Upper Limb MCQs

अब Quizwiz के साथ अपने होमवर्क और परीक्षाओं को एस करें!

B. Ginglymus joint is the correct technical term to describe a hinge joint. It allows motion in one axis (flexion and extension in the case of the humeroulnar joint) and is therefore a uniaxial joint. The other types of joints listed allow motion in more than one axis.

A 29-year-old patient has a dislocated elbow in which the ulna and medial part of the distal humerus have become separated. What classification of joint is normally formed between these two bones? A. Trochoid B. Ginglymus C. Enarthrodial D. Synarthrosis E. Sellar

B. During a breech delivery as described here, downward traction is applied to the shoulders and upper limbs as the baby is forcibly extracted from the birth canal. This exerts traction on the upper cord of the brachial plexus, often causing a traction injury from which the baby can often recover. If the ventral rami of C5 and C6 are avulsed from the spinal cord, the injury is permanent

A 22-year-old pregnant woman was admitted urgently to the hospital after her baby had begun to appear at the introitus. The baby had presented in the breech position, and it had been necessary to exert considerable traction to complete the delivery. The newborn is shown in Fig. 6-7. Which of the following structures was most likely injured by the trauma of childbirth? A. Radial nerve B. Upper trunk of the brachial plexus C. Lower trunk of the brachial plexus D. Median, ulnar, and radial nerves E. Upper and lower trunks of the brachial plexus

A. The nerve most likely affected is the radial nerve as it travels in the radial groove as it descends along the humerus to the forearm. The axillary nerve runs around the surgical neck of the humerus, the median nerve passes superficial to the muscles of the arm, the ulnar nerve runs at the medial epicondyle, and the musculocutaneous nerve pierces the coracobrachialis muscle and then runs

A 25-year-old man falls on a slippery trail and injures his upper limb. Inspection reveals abrasions over his arm. A radiograph shows a fracture at the radial groove of the humerus. Which of the following nerves will most likely be injured? A. Radial B. Axillary C. Median D. Ulnar E. Musculocutaneous

E. The common flexor sheath encloses the long flexor tendons of the fingers in the carpal tunnel and proximal palm. This sheath is usually continuous with the flexor sheath of the little (fifth) finger, which continues within the palm, having no connection with sheaths of the other digits, which do not extend into the digits.

A 10-year-old boy suffered a dog bite that entered the common flexor synovial sheath of his forearm. He was admitted to the hospital, where the wound was cleaned and dressed and he was treated further with rabies antiserum. Two days later the boy had an elevated temperature and his palm and one digit were obviously swollen, causing him to cry with pain. Into which of the digits could the infection spread most easily, following the anatomy of the typical common flexor sheath? A. First B. Second C. Third D. Fourth E. Fifth

B. The abductor pollicis longus and extensor pollicis brevis muscles are the occupants of the first dorsal compartment of the wrist. The extensor carpi radialis longus and brevis are in the second compartment. The extensor digitorum is in the third compartment, as is the extensor indicis. The extensor carpi ulnaris is located in the sixth dorsal compartment.

A 15-year-old girl was brought to the emergency department with a tear of the tendons in the first dorsal compartment of the wrist from a severe bite by a pit bull dog. The injured tendons in this compartment would include which of the following muscles? A. Extensor carpi radialis longus and brevis B. Abductor pollicis longus and extensor pollicis brevis C. Extensor digitorum D. Extensor indicis proprius E. Extensor carpi ulnaris

E. Striking the concrete blocks with the medial side of her hand has injured the ulnar nerve in Guyon's canal. This is the triangular tunnel formed by the pisiform bone medially, the flexor retinaculum dorsally, and the deep fascia of the wrist ventrally. This injury would result in loss of sensation to the medial palm and the palmar surface of the medial one and a half digits and motor loss of the hypothenar muscles, the interossei, and the medial two lumbricals. The median nerve is not involved because the thenar muscles and lateral palmar sensations are intact. The dorsal ulnar nerve arises proximal to the wrist, thus it would not be lost. Carpal bone dislocation is unlikely. If the lunate bone were dislocated, it would not cause compression of the ulnar nerve at the wrist. There is no indication of fifth metacarpal fracture, the so-called boxer's fracture.

A 17-year-old female student of martial arts entered the emergency department with a complaint of pain in her hand. Patient history reveals that she had been breaking concrete blocks with her hand. Examination reveals that the patient has weak abduction and adduction of her fingers but has no difficulty in flexing them. The patient also has decreased sensation over the palmar surfaces of the fourth and fifth digits. Which of the following best describes the nature of her injury? A. Compression of the median nerve in the carpal tunnel B. Fracture of the triquetrum, with injury to the dorsal ulnar nerve C. Dislocation of a bone in the proximal row of the carpus D. Fracture of the shaft of the fifth metacarpal E. Injury of the ulnar nerve in Guyon's canal

C. The radial nerve acts to extend the forearm at the elbow. This nerve is derived from all the ventral rami of the brachial plexus C5 to T1. None of the other answers includes all the ventral rami and are therefore incorrect.

A 22-year-old man is admitted to the hospital after a car collision. Radiographic examination reveals an oblique fracture of his humerus. Upon physical examination the patient is unable to extend his forearm. The damaged nerve was most likely composed of fibers from which of the following spinal levels? A. C5, C6 B. C5, C6, C7 C. C5, C6, C7, C8, T1 D. C6, C7, C8, T1 E. C7, C8, T1

C. The injury is at the second part of the axillary artery. The suprascapular artery is a branch of the thyrocervical trunk off the subclavian artery, proximal to the axillary artery. The subscapular artery is the major branch of the third part of the axillary artery, giving off the thoracodorsal and the circumflex scapular arteries. In this case blood would be flowing from the circumflex scapular artery in a retrograde direction into the axillary artery, supplying blood distal to the injury.

A 22-year-old woman had suffered a severe knife wound to the upper lateral portion of her pectoral region, with entry of the knife at the deltopectoral groove. Pressure applied to the wound had prevented further profuse bleeding. In the emergency department, vascular clamps were applied to the axillary artery, proximal and distal to the site of injury, which had occurred between the second and third parts of the axillary artery. The vascular surgeon knew there was time to repair the wound of the artery because of the rich collateral pathway provided by the anastomoses between which of the following arteries? A. Transverse cervical and suprascapular B. Posterior circumflex humeral and profunda brachii C. Suprascapular and circumflex scapular D. Supreme (superior) thoracic and thoracoacromial E. Lateral thoracic and suprascapular

D. The location for palpation of the radial pulse is lateral to the tendon of the flexor carpi radialis, where the radial artery can be compressed against the distal radius. The radial pulse can also be felt in the anatomic snuffbox between the tendons of the extensor pollicis brevis and extensor pollicis longus muscles, where the radial artery can be compressed against the scaphoid.

A 22-year-old woman is admitted to the emergency department in an unconscious state. The nurse takes a radial pulse to determine the heart rate of the patient. This pulse is felt lateral to which tendon? A. Palmaris longus B. Flexor pollicis longus C. Flexor digitorum profundus D. Flexor carpi radialis E. Flexor digitorum superficialis

C. The patient is suffering from thoracic outlet syndrome, involving neural and vascular elements. This results from any condition that decreases the dimensions of the superior thoracic aperture (the formal name of the thoracic outlet). It could be a result of a cervical rib, accessory muscles, and/or atypical connective tissue bands at the root of the neck. In this case, symptoms involve the arm, forearm, and hand. Paraesthesia along the medial forearm and hand and atrophy of long flexors and intrinsic muscles point to a possible compression or traction problem of the lower trunk (C8, T1) rather than a lesion of either the median or ulnar nerve. The lateral palm has no sensory problem, which tends to rule out median nerve involvement. Changes in the radial artery pulse point to vascular compression. Erb-Duchenne paralysis of the upper trunk would affect proximal limb functions, such as arm rotation, abduction, and so on. This lesion is on the left side, so the brachiocephalic artery could not be involved because it arises from the right side of the aortic arch; moreover, it would not compress the brachial plexus. Carpal tunnel syndrome would not explain the problems of the forearm and medial hand, or the long flexor atrophy. An isolated medial cord lesion would not explain the atrophy of all long flexors and intrinsic muscles and does not explain the radial pulse characteristics. The ischemic pain in the arm is due to vascular compression.

A 22-year-old woman visits the outpatient clinic with pain in her left upper limb. She has a long history of pain in this limb and difficulty with fine motor tasks of the hand. Physical examination reveals paraesthesia along the medial surface of the forearm and palm and weakness and atrophy of gripping muscles ("long flexors") and the intrinsic muscles of the hand. The radial pulse is diminished when her neck is rotated to the ipsilateral side (positive Adson's test). What is the most likely diagnosis? A. Erb-Duchenne paralysis B. Aneurysm of the brachiocephalic artery, with plexus compression C. Thoracic outlet syndrome D. Carpal tunnel syndrome E. Injury to the medial cord of the brachial plexus

A. A synarthrosis joint is a fibrous connection that allows minimal to no movement. In this case, virtually no movement is allowed by the interosseous membrane joint between the radius and ulna. Symphysis joints are permanent fibrocartilaginous fusions between two bones; pubic symphysis is an example. Synchondrosis is a temporary joint made of cartilage that transitions to bone typically after growth completes (i.e., epiphyseal plate). Trochoid joints are pivot joints, and the humeral-radial portion of the elbow joint is an example. Ginglymus joints are hinge joints located at the interphalangeal junctions in the hand and foot (PIPs and DIPs).

A 23-year-old woman arrives at the emergency department with a swollen, painful forearm. An MRI examination reveals a compartment syndrome originating at the interosseous membrane between the radius and ulna. Which of the following type of joint will most likely be affected? A. Synarthrosis B. Symphysis C. Synchondrosis D. Trochoid E. Ginglymus

D. Interestingly, "gamekeeper's thumb" was a term coined because this injury was most commonly associated with Scottish gamekeepers who, it is said, killed small animals such as rabbits by breaking their necks between the ground and the gamekeeper's thumb and index finger. The resulting valgus force on the abducted metacarpophalangeal (MCP) joint caused injury to the ulnar collateral ligament. These days this injury is more commonly seen in skiers who land awkwardly with their hand braced on a ski pole, causing the valgus force on the thumb as is seen in this patient. Whereas the term "skier's thumb" is sometimes used, "gamekeeper's thumb" is still in common usage.

A 23-year-old woman had a painful injury to her hand in a dry ski-slope competition, in which she fell and caught her thumb in the matting. Radiographic and physical examinations reveal rupture of the ulnar collateral ligament of the metacarpophalangeal joint of the thumb. Lidocaine is injected into the area to relieve the pain, and she is scheduled for a surgical repair. From which of the following clinical problems is she suffering? A. De Quervain's syndrome B. Navicular bone fracture C. Boxer's thumb D. Gamekeeper's thumb E. Bennett's thumb

A. The axillary nerve is a direct branch of the posterior cord and wraps around the surgical neck of the humerus to innervate the teres minor and the deltoid muscles. With this anatomic arrangement, the axillary nerve is tightly "tethered" to the proximal humerus. When the head of the humerus is dislocated, it often puts traction on the axillary nerve.

A 24-year-old female basketball player is admitted to the emergency department after an injury to her shoulder. Radiographic examination reveals a shoulder dislocation. What is the most commonly injured nerve in shoulder dislocations? A. Axillary B. Radial C. Median D. Ulnar E. Musculocutaneous

B. The anatomic snuffbox is formed by the tendons of the extensor pollicis brevis, the abductor pollicis longus, and the extensor pollicis longus. The floor is formed by the scaphoid bone, and it is here that one can palpate for a possible fractured scaphoid.

A 25-year-old male athlete is admitted to the emergency department after a bad landing while performing the pole vault. Radiographic examination of his hand reveals a fractured carpal bone in the floor of the anatomic snuffbox (Fig. 6-2). Which bone has most likely been fractured? A. Triquetral B. Scaphoid C. Capitate D. Hamate E. Trapezoid

C. This type of dislocation is common in children and results when the radius is dislocated and slips out from the anular ligament, which holds it in place, articulating with the ulna and the capitulum of the humerus. In adults the anular ligament has a good "grip" at the radial neck, but in young children the radial head is not fully developed, leading to an indistinct neck. Compression of the median nerve is not likely due to its medial position in the cubital fossa. The radius does not articulate with the trochlea of the humerus; the ulna articulates at this position. The ulna is not likely to be dislocated because it is more stable than the radius, which has only the anular ligament for its support. The radial nerve does not pass behind the medial epicondyle; rather, the ulnar nerve does this, so this is not the correct answer.

A 3-year-old girl is admitted to the emergency department with severe pain. History taking reveals that the girl was violently lifted by her raised arm by her mother to prevent the girl from walking in front of a moving car. Which of the following is most likely the cause of the pain? A. Compression of the median nerve B. Separation of the head of the radius from its articulation with the trochlea of the humerus C. Separation of the head of the radius from its articulation with the ulna and the capitulum of the humerus D. Separation of the ulna from its articulation with the trochlea of the humerus E. Stretching of the radial nerve as it passes behind the medial epicondyle of the humerus

B. The patient can extend his forearm, which suggests that the triceps brachii muscle is not weakened. Supination appears to be weak along with hand grasp and wrist drop. This would indicate that part of the radial nerve has been lost below the innervation of the triceps brachii and above the branches to the supinator and extensors in the forearm. However, sensation on the forearm and hand is intact, indicating that the superficial branch of the radial nerve is intact. The superficial branch of the radial nerve separates from the deep radial nerve at the distal third of the humerus. The posterior cord of the brachial plexus is responsible for providing innervation of the axially and radial nerves. This patient does have some radial nerve innervation and no loss of axillary nerve function. The patient does not have weakened adduction of the wrist, indicating that the ulnar nerve is not injured. If both the radial and musculocutaneous nerves are injured, supination would not be possible as the supinator and biceps brachii muscles provide supination of the forearm.

A 32-year-old man is admitted to the emergency department after a severe car crash. Radiographic examination reveals multiple fractures of his right upper limb. A surgical procedure is performed and metallic plates are attached to various bony fragments to restore the anatomy. Five months postoperatively the patient visits the outpatient clinic. Upon physical examination the patient can abduct his arm and extend the forearm, and the sensation of the forearm and hand is intact; however, hand grasp is very weak, and he cannot extend his wrist against gravity. Which of the following nerves was most likely injured during the surgical procedure? A. Posterior cord of the brachial plexus B. Radial nerve at the distal third of the humerus C. Radial and ulnar D. Radial, ulnar, and median E. Radial and musculocutaneous

D. A cervical rib (usually found at C7) may cause thoracic outlet syndrome, which is a condition characterized by weak muscle tone in the hand and loss of radial pulse when the upper limb is abducted above the shoulder. The mechanism of injury with the gun being fired overhead suggests a lower trunk injury to the brachial plexus. The axillary artery supplies the shoulder muscles, and there is no loss of function to these muscles. The upper trunk of the brachial plexus also supplies innervation to the shoulder muscles, which are unaffected based on the patient's presenting abnormalities. The subclavian artery is located anterior to the brachial plexus until the plexus separates into cords as it passes under the clavicle. The brachiocephalic artery and lower trunk of the brachial plexus is only partially correct; the brachiocephalic artery is not directly associated with the brachial plexus due to its location at the midline of the body behind the sternum.

A 32-year-old man who is an expert target shooter reports pain in his right upper limb and slight tingling and numbness of all digits of the ipsilateral hand. However, the tingling and numbness of the fourth and fifth digits is the most severe. The man states that the problem usually occurs when he is firing his gun with his hand overhead. Radiographic studies reveal the presence of a cervical rib and accessory scalene musculature. Which of the following structures is most likely being compressed? A. Axillary artery B. Upper trunk of brachial plexus C. Subclavian artery D. Lower trunk of brachial plexus E. Brachiocephalic artery and lower trunk of brachial plexus

A. The ulnar nerve innervates the dorsal and palmar interossei, which act to abduct and adduct the fingers and assist the lumbricals in their actions of flexing the metacarpophalangeal joints and extending the interphalangeal joints. The recurrent branch of the median nerve innervates the thenar muscle group that functions in the movement of the thumb. The radial and musculocutaneous nerves do not innervate any muscles in the hand. The anterior interosseous innervates the flexor pollicis longus and the pronator quadratus.

A 34-year-old man is admitted to the hospital after a car collision. Radiographic examination reveals a fracture at his wrist. Physical examination reveals paralysis of the muscles that act to extend the interphalangeal joints (Fig. 6-9). Which of the following nerves is most likely injured? A. Ulnar B. Recurrent branch of median C. Radial D. Musculocutaneous E. Anterior interosseous

A. The superior trunk of the brachial plexus includes C5 and C6, which give rise to the suprascapular nerve, which innervates the supraspinatus muscle. The supraspinatus muscle is the primary muscle involved in abduction of the arm from 0 to 15 degrees. The deltoid muscle, supplied primarily by C5, abducts the arm from 15 to 90 degrees. The middle trunk is just C7 and has nothing to do with the muscle involved in initial abduction of the arm. The inferior trunk is C8-T1 and does not supply the supraspinatus muscle; therefore, it is not the right answer. The cords are distal to the branching of the supraspinatus muscle; therefore, neither lateral cord nor medial cord is the correct answer.

A 34-year-old woman is admitted to the emergency department after a car crash. Radiographic studies show marked edema and hematoma of the arm, but there are no fractures. During physical examination the patient presents with inability to abduct her arm without first establishing lateral momentum of the limb, and inability to flex the elbow and shoulder. Which of the following portions of the brachial plexus is most likely injured? A. Superior trunk B. Middle trunk C. Inferior trunk D. Lateral cord E. Medial cord

A. Axillary-subclavian vein thrombosis is becoming much more common in recent years because of the extensive use of catheters in cancer patients and other chronic medical conditions. Effort-induced thrombosis is seen with strenuous use of the dominant arm with hyperabduction and external rotation of the arm or backward and downward rotation of the shoulder as in playing cricket, volleyball, or baseball, or chopping wood. Because the symptoms of subclavian stenosis are fairly dramatic, most patients present promptly, usually within 24 hours. They complain of a dull ache in the shoulder and axilla, the pain worsened by activity. Conversely, rest and elevation often relieve the pain. Patients with catheter-associated axillary-subclavian deep vein thrombosis report similar symptoms at the arm or shoulder on the side with the indwelling catheter.

A 36-year-old man is admitted to the emergency department with a dull ache in his shoulder and axilla (Fig. 6-8). During physical examination the pain worsens by activity, and, conversely, rest and elevation relieve the pain. History reveals that the patient was hospitalized 2 days ago and a central venous line was used. What is the most likely diagnosis? A. Axillary-subclavian vein thrombosis B. Compression of C5 to C8 spinal nerve C. Disc herniation of C4 to C8 D. Impingement syndrome E. Injury to radial, ulnar, and median nerves

C. The triceps brachii muscle is innervated by the radial nerve (primarily C7), which comes off C5 to T1 spinal nerves. Because the patient's only motor deficit involves the triceps brachii muscles, one can rule out C5 and C6, which supply fibers to the axillary, musculocutaneous, and upper subscapular nerves. Damage to either of these ventral rami would result in additional motor deficits of the shoulder and flexor compartment of the arm. One can also rule out C8-T1 because these ventral rami form the medial pectoral nerve and the medial brachial and antebrachial cutaneous nerves. Damage to these ventral rami would result in loss of pectoral muscle function and cutaneous sensation over the medial surface of the upper limb.

A 45-year-old woman is admitted to the hospital with neck pain. An MRI examination reveals a herniated disc in the cervical region. Physical examination reveals weak triceps brachii muscle. Which of the following spinal nerves is most likely injured? A. C5 B. C6 C. C7 D. C8 E. T1

Answer D. Suspensory (Cooper's) ligaments The suspensory ligaments of the breast, also known as Cooper's ligaments, are fibrous bands that run from the dermis of the skin to the deep layer of superficial fascia and are primary supports for the breasts against gravity. Ptosis of the breast is usually due to the stretching of these ligaments and can be repaired with plastic surgery. Scarpa's fascia is the deep membranous layer of superficial fascia of the anterior abdominal wall. The pectoralis major and pectoralis minor are muscles that move the upper limb and lie deep to the breast but do not provide any direct support structure to the breast. The serratus anterior muscle is involved in the movements of the scapula.

A 45-year-old woman is being examined as a candidate for cosmetic breast surgery. The surgeon notes that both of her breasts sag considerably. Which structure(s) has most likely become stretched to result in this condition? A. Scarpa's fascia B. Pectoralis major muscle C. Pectoralis minor muscle D. Suspensory (Cooper's) ligaments E. Serratus anterior muscle

E. During a fall on an outstretched upper limb, the forces are conducted through the hand on up through the bones of the limb in succession. Often these bones do not fracture but rather pass the compressive forces proximally. The appendicular skeleton joins with the axial skeleton at the sternoclavicular joint. The forces are not sufficiently transferred to the sternum, causing the clavicle to absorb the force, resulting in common pediatric fracture of this sigmoidal-shaped bone.

A 5-year-old boy is admitted to the emergency department after falling from a tree. The parents are informed by the radiologist that their son's fracture is the most common fracture that occurs in children. Which of the following bones was broken? A. Humerus B. Radius C. Ulna D. Scaphoid E. Clavicle

B. The posterior interosseous nerve is an extension of the deep branch of the radial nerve after it emerges distal to the supinator. It is responsible for innervation of several muscles in the extensor compartment of the posterior aspect of the forearm, including extension of the metacarpophalangeal joints. The deep radial nerve courses laterally around the radius and passes between the two heads of the supinator muscle and is thus likely to be compressed by a hematoma between the fractured radius and the supinator muscle. Though the radial nerve gives rise to the posterior interosseous nerve, this answer choice is too general and would not indicate the precise injured branch of the radial nerve. Both the deep branch of the ulnar nerve and the median nerve traverse the medial and anteromedial aspect of the arm, respectively. These nerves primarily supply the flexor compartment of the arm. The anterior interosseous nerve is a branch of the median nerve and supplies the flexor digitorum profundus, flexor pollicis longus, and the pronator quadratus muscles

A 52-year-old man is admitted to the emergency department after falling on wet pavement. Radiographic examination reveals fracture of the radius. An MRI study reveals a hematoma between the fractured radius and supinator muscle. Upon physical examination the patient has weakened abduction of the thumb and extension of the metacarpophalangeal joints of the fingers. Which of the following nerves is most likely affected? A. Anterior interosseous B. Posterior interosseous C. Radial nerve D. Deep branch of ulnar nerve E. Median nerve

C. The winged scapula results from a lesion of the long thoracic nerve, which supplies the serratus anterior muscle. This muscle is responsible for rotating the scapula upward, which occurs during abduction of the arm above the horizontal. The long thoracic nerve arises from the ventral rami of C5 to C7 of the brachial plexus. The diaphragm is supplied by the phrenic nerve, which comes from the ventral rami of C3 to C5 (mnemonic: C3, 4 and 5 keep the diaphragm alive).

A 55-year-old male firefighter is admitted to the hospital after blunt trauma to his right axilla. Examination reveals winging of the scapula and partial paralysis of the right side of the diaphragm. Which of the following parts of the brachial plexus have been injured? A. Cords B. Divisions C. Ventral rami D. Terminal branches E. Trunks

B. The lunate bone is the most commonly dislocated carpal bone. Displacement is almost always anteriorly. Dislocation of the lunate bone can precipitate the signs associated typically with carpal tunnel syndrome.

A 54-year-old woman presents with pain in her right wrist that resulted when she fell forcefully on her outstretched hand. Radiographic studies indicate an anterior dislocation of a carpal bone of the proximal row (see Fig. 6-4). Which of the following bones is most commonly dislocated? A. Capitate B. Lunate C. Scaphoid D. Pisiform E. Triquetrum

A. The suprascapular artery arises as a major branch of the thyrocervical trunk from the subclavian artery. It has rich anastomoses with the circumflex scapular artery and could provide essential blood supply to the scapula. The dorsal scapular artery would be lost with the graft. None of the other vessels listed is in position to provide adequate supply to the scapula.

A 56-year-old woman is admitted to the hospital after a severe car crash. A large portion of her chest wall needed to be surgically removed and replaced with a musculo-osseous scapular graft involving the medial border of the scapula. Which of the following arteries will most likely recompensate the blood supply to the entire scapula? A. Suprascapular B. Dorsal scapular artery C. Posterior circumflex humeral artery D. Lateral thoracic E. Supreme thoracic artery

C. The scaphoid and lunate carpal bones have a direct articulation with the radius, which is fractured in a Colles' fracture; therefore, they would most likely be disrupted or fractured. The other carpal bones listed do not have direct contact with the radius and have a more distal location; therefore, they would not be as likely to be injured with a Colles' fracture.

A 56-year-old woman visits the emergency department after falling on a wet pavement. Radiographic examination reveals osteoporosis and a Colles' fracture. Which of the following carpal bones are often fractured or dislocated with a Colles' fracture? A. Triquetrum and scaphoid B. Triquetrum and lunate C. Scaphoid and lunate D. Triquetrum, lunate, and scaphoid E. Triquetrum and pisiform

B. The radial artery enters the palm through the anatomic snuffbox. The artery then moves on to pierce through the two heads of the first dorsal interosseous muscle and enter the deep aspect of the palm. The flexor pollicis longus tendon runs on the palmar aspect of the hand and the radial artery runs on the dorsal aspect of the hand before entering the deep aspect of the palm, and therefore the radial artery does not run below this tendon. The radial artery does not run between the first and second interosseous muscle and therefore cannot be used as a landmark to identify the artery. Finally, the artery does not run between the first dorsal interosseous muscle and the adductor pollicis longus.

A 59-year-old woman is admitted to the hospital in a state of shock. During physical examination, several lacerations are noted in her forearm and her radial pulse is absent. Where is the most typical place to identify the radial artery immediately after crossing the radiocarpal joint? A. Between the two heads of the first dorsal interosseous muscle B. In the anatomic snuffbox C. Below the tendon of the flexor pollicis longus D. Between the first and second dorsal interossei muscles E. Between the first dorsal interosseous muscle and the adductor pollicis longus

C. Injury to the radial nerve can be caused by a blow to the midhumeral region because the nerve winds around the shaft of the humerus. The symptoms described include the loss of wrist and finger extension and a loss of sensation in an area of skin supplied by the radial nerve.

A 61-year-old man was hit by a cricket bat in the midhumeral region of his left arm. Physical examination reveals normal elbow motion; however, he could not extend his wrist or his metacarpophalangeal joints and he reported a loss of sensation on a small area of skin on the dorsum of the hand proximal to the first two digits. Radiographic examination reveals a hairline fracture of the shaft of the humerus just distal to its midpoint. Which of the following nerves is most likely injured? A. Median B. Ulnar C. Radial D. Musculocutaneous E. Axillary

A. The median nerve supplies sensory innervation to the thumb, index, and middle fingers as well as to the lateral half of the ring finger. The median nerve also provides motor innervation to muscles of the thenar eminence. Compression of the median nerve in the carpal tunnel explains these deficits in conjunction with normal functioning of the flexor compartment of the forearm. The ulnar nerve is not implicated in these symptoms. Compression of the brachial plexus could not be attributed to pressure from hypertrophy of the triceps brachii muscle, it is located distal to the plexus. In addition, symptoms would include several upper limb deficits rather than the focal symptoms described in this instance. Osteoarthritis of the cervical spine would also lead to increasing complexity of symptoms.

A 69-year-old man has numbness and pain in the middle three digits of his right hand at night. He retired 9 years ago after working as a carpenter for 30 years. He has atrophy of the thenar eminence (see Fig. 6-3). Which of the following conditions will be the most likely cause of this atrophy? A. Compression of the median nerve in the carpal tunnel B. Formation of the osteophytes that compress the ulnar nerve at the ulnar condyle C. Hypertrophy of the triceps brachii muscle compressing the brachial plexus D. Osteoarthritis of the cervical spine E. Repeated trauma to the ulnar nerve

D. The median nerve provides innervation to most of the muscles in the flexor compartment of the forearm; cutaneous innervation of the second, third, and fourth digits and palmar and dorsum aspects of the hand; and innervation of four intrinsic hand muscles: first and second lumbricals, abductor pollicis brevis, opponens pollicis, and flexor pollicis brevis. The thenar compartment contains the abductor pollicis brevis, opponens pollicis, and flexor pollicis brevis muscles, and these muscles are innervated by the recurrent branch of the median nerve. The patient has weakening of the first two lumbricals and not simply the thenar muscles, so the median nerve is most likely to be compressed. Another indication that the median nerve is compressed is the vigorous shaking of the wrist. Because the median nerve traverses the carpal tunnel, carpal tunnel compression could lead to this action on part of the patient. The ulnar nerve provides innervation for part of the flexor digitorum profundus and flexor carpi ulnaris muscles. These muscles are not weakened in this patient. The radial nerve provides cutaneous supply to the dorsum of the hand and forearm as well as extensor muscles of the forearm. The posterior interosseous nerve is a branch of the radial nerve and provides innervation of the extensor muscles in the forearm

A 69-year-old woman visits the outpatient clinic with a complaint of numbness and tingling of her hand for the past 3 months. Physical examination reveals she has numbness and pain in the lateral three digits of her right hand that are relieved by vigorous shaking of the wrist. In addition, the abductor pollicis brevis, opponens pollicis, and the first two lumbrical muscles are weakened. Sensation was decreased over the lateral palm and the volar aspect of the first three digits. Which of the following nerves is most likely compressed? A. Ulnar B. Radial C. Recurrent median D. Median E. Posterior interosseous

C. The supraspinatus muscle inserts on the greater tubercle of the humerus and is said to initiate abduction of the arm at the shoulder. It is supplied principally by spinal nerve C5. The subscapularis muscle is the only muscle that inserts on the lesser tubercle. The subscapularis muscle is innervated by the upper and lower subscapular nerves. The teres minor muscle takes origin from the lateral border of the scapula; the teres major muscle takes origin from the region of the inferior angle and the lateral border of the scapula.

After an orthopedic surgeon examined the MRI of the shoulder of a 42-year-old woman he informed her that the supraspinatus tendon was injured and needed to be repaired surgically. Which of the following is true of the supraspinatus muscle? A. It inserts onto the lesser tubercle of the humerus. B. It initiates adduction of the shoulder. C. It is innervated chiefly by the C5 spinal nerve. D. It is supplied by the upper subscapular nerve. E. It originates from the lateral border of the scapula.

A. The tendon of the long head of the biceps brachii muscles runs in the intertubercular groove on the proximal humerus as it changes direction and turns medially to attach to the supraglenoid tubercle of the scapula. This change in direction within an osseous structure predisposes the tendon to wear and tear, particularly in people who overuse the biceps brachii muscle. This type of injury presents with a characteristic sign called the "Popeye sign" after the cartoon character.

An 85-year-old man is admitted to the hospital with a painful arm after lifting a case of wine. Physical examination gives evidence of a rupture of the long tendon of the biceps brachii (Fig. 6-6). Which of the following is the most likely location of the rupture? A. Intertubercular groove B. Midportion of the biceps brachii muscle C. Junction with the short head of the biceps brachii muscle D. Proximal end of the combined biceps brachii muscle E. Bony insertion of the muscle

D. The left spinal accessory nerve (CN XI) has been injured distal to the sternocleidomastoid muscle, resulting in paralysis of the trapezius, allowing the shoulder to droop and the superior angle to push out posteriorly. The sternocleidomastoid muscles are intact, as demonstrated by symmetry in strength in turning the head to the right and left. There is no indication of paralysis of the lateral rotators of the shoulder or elbow flexors (suprascapular nerve or upper trunk). Thoracodorsal nerve injury would result in paralysis of the latissimus dorsi muscle, an extensor, and medial rotator of the humerus.

During a fight in a tavern, a 45-year-old male construction worker received a shallow stab wound from a broken beer bottle at a point near the middle of the left posterior triangle of his neck. Upon physical examination, it is observed that the left shoulder is drooping lower than the right shoulder, and the superior angle of the scapula juts out slightly. Strength in turning the head to the right or left appears to be symmetric. Which of the following nerves is most likely injured? A. Suprascapular nerve in the supraspinous fossa B. The terminal segment of the dorsal scapular nerve C. The upper trunk of the brachial plexus D. The spinal accessory nerve in the posterior cervical triangle E. The thoracodorsal nerve in the axilla

A. Injury to the radial nerve in the spiral groove will paralyze the abductor pollicis longus muscle and both extensors of the thumb. This injury will also lead to wrist drop (inability to extend the wrist). Weakness of grip would also occur, although this is not mentioned in the question. If the wrist is flexed, finger flexion and grip strength are weakened because the long flexor tendons are not under tension. Note how much your strength of grip is increased when your wrist is extended versus when it is flexed.

Examination of a 21-year-old female athlete with an injury of the radial nerve in the spiral groove would typically demonstrate which of the following physical signs? A. Weakness of thumb abduction and thumb extension B. Weakness of thumb opposition C. Inability to extend the elbow D. Paralysis of pronation of the hand E. Paralysis of abduction and adduction of the arm

A. This proximal injury to the median nerve would paralyze all of the long flexors of the digits, except for the muscle that flexes the distal interphalangeal joints of digits 4 and 5, thereby swinging the "balance of power" to the muscles that extend the digits, all of which are innervated by the radial nerve. The intrinsic hand muscles can aid in flexion of the metacarpophalangeal joints, and are innervated by the ulnar nerve. However, they are of insufficient size to compensate for the extensor forces exerted on fingers.

In a penetrating wound to the forearm of a 24-year-old man, the median nerve is injured at the entrance of the nerve into the forearm. Which of the following would most likely be apparent when the patient's hand is relaxed? A. The MCP and IP joints of the second and third digits of the hand will be in a condition of extension. B. The third and fourth digits will be held in a slightly flexed position. C. The thumb will be flexed and slightly abducted. D. The first, second, and third digits will be held in a slightly flexed position. E. The MCP and IP joints of the second and third digits of the hand will be in a condition of flexion.

Answer A. The three chief contents of the cubital fossa are the biceps brachii tendon, brachial artery, and median nerve (lateral to medial). The common and anterior interosseous arteries arise distal to the cubital fossa; the ulnar and radial arteries are the result of the bifurcation of the brachial artery distal to the cubital fossa.

Laboratory studies in the outpatient clinic on a 24-year-old woman included assessment of circulating blood chemistry. Which of the following arteries is most likely at risk during venipuncture at the cubital fossa? A. Brachial B. Common interosseous C. Ulnar D. Anterior interosseous E. Radial

A. The ulnar nerve enters the forearm by passing between the two heads of the flexor carpi ulnaris and descends between and innervates the flexor carpi ulnaris (for medial wrist deviation) and flexor digitorum profundus (medial half) muscles. Injuring the ulnar nerve results in claw hand. It enters the hand superficial to the flexor retinaculum and lateral to the pisiform bone, where it is vulnerable to damage. The ulnar nerve also enters Guyon's canal, but damage to it here would not present with the aforementioned symptoms. The median nerve enters the carpal tunnel and the radial nerve passes deep to the brachioradialis.

Physical examination reveals weakness of medial deviation of the wrist (adduction), loss of sensation on the medial side of the hand, and clawing of the fingers. Where is the most likely place of injury? A. Compression of a nerve passing between the humeral and ulnar heads of origin of flexor carpi ulnaris B. Compression of a nerve passing at Guyon's canal between the pisiform bone and flexor retinaculum C. Compression of a nerve passing through the carpal tunnel D. Compression of a nerve passing between the ulnar and radial heads of origin of flexor digitorum superficialis E. Compression of a nerve passing deep to brachioradialis muscle

C. The infectious agent was introduced into the synovial sheath of the long tendons of the little (fifth) finger. Proximally, this sheath runs through the midpalmar space, and inflammatory processes typically rupture into this space unless aggressively treated with the appropriate antibiotics

While sharpening his knife, a 23-year-old male soldier accidentally punctured the ventral side of the fifth digit at the base of the distal phalanx. The wound became infected, and within a few days the infection had spread into the palm, within the sheath of the flexor digitorum profundus tendons. If the infection were left untreated, into which of the following spaces could it most likely spread? A. Central compartment B. Hypothenar compartment C. Midpalmar space D. Thenar compartment E. Thenar space

B. The radial pulse is best located on the forearm (antebrachium) just proximal to the wrist joint. At this point the radial artery travels on the distal radius between the flexor carpi radialis and brachioradialis tendons. The palmaris longus tendon travels more medially to the radial artery and above the flexor retinaculum. The flexor pollicis longus tendon is a deeper structure in the antebrachium and is also located medially to the radial artery.

While working out with weights, a 28-year-old woman experiences a severe pain in her chest. The pain is referred to the anterior chest wall, radiating to the mandible and her left arm. The woman felt dizzy and after 10 minutes she collapsed and was unconscious. A physician happened to be near the woman and immediately tried to feel her radial pulse. The radial artery lies between two tendons near the wrist, which are useful landmarks. Which of the following is the correct pair of tendons? A. Flexor carpi radialis and palmaris longus B. Flexor carpi radialis and brachioradialis C. Brachioradialis and flexor pollicis longus D. Flexor pollicis longus and flexor digitorum superficialis E. Flexor pollicis longus and flexor digitorum profundus

A. The palmaris longus passes along the midline of the flexor surface of the forearm. The flexor carpi radialis is seen in the lateral portion of the forearm superficially, passing over the trapezium to insert at the base of the second metacarpal. The abductor pollicis longus tendon is laterally located in the wrist, where it helps form the lateral border of the anatomic snuffbox. The flexor carpi ulnaris tendon can be seen and palpated on the medial side of the wrist ventrally. The flexor pollicis longus tendon passes deep through the carpal tunnel.

A 12-year-old boy lacerated the palmar surface of the wrist while playing with a sharp knife. The cut ends of a tendon could be seen within the wound in the exact midline of the wrist. Which tendon lies in this position in most people? A. Palmaris longus B. Flexor carpi radialis C. Abductor pollicis longus D. Flexor carpi ulnaris E. Flexor pollicis longus

C. The radial nerve innervates the extensor compartments of the arm and the forearm. It supplies the triceps brachii proximal to the spiral groove, so elbow extension is intact here. It also provides sensory innervation to much of the posterior arm and forearm as well as the dorsal thumb, index, and middle fingers up to the level of the fingernails. Symptoms are described only in the distal limb due to the midhumeral location of the lesion. The median nerve innervates flexors of the forearm and thenar muscles and provides sensory innervation to the lateral palmar hand. The ulnar nerve supplies only the flexor carpi ulnaris and the medial half of the flexor digitorum profundus in the forearm. Additionally, its sensory distribution is to both the palmar and dorsal aspects of the medial hand. It does not supply extensor muscles.

A 13-year-old boy is brought to the emergency department after losing control during a motorbike race in which he was hit by several of the other racers. Physical examination reveals several cuts and bruises. He is unable to extend the left wrist, fingers, and thumb, although he can extend the elbow. Sensation is lost in the lateral half of the dorsum of the left hand. Which of the following nerves has most likely been injured to result in these signs, and in what part of the arm is the injury located? A. Median nerve, anterior wrist B. Median nerve, arm C. Radial nerve, midhumerus D. Ulnar nerve, midlateral forearm E. Ulnar nerve, midpalmar region

C. Trauma both to the median and ulnar nerves at the wrist results in total clawing of the fingers. The metacarpophalangeal joints of all digits are extended by the unopposed extensors because the radial nerve is intact. All interossei and lumbricals are paralyzed because the deep branch of the ulnar nerve supplies all of the interossei; lumbricals I and II are paralyzed, for they are innervated by the median nerve; lumbricals III and IV are paralyzed, for they receive supply from the deep ulnar nerve. The interossei and lumbricals are responsible for extension of the interphalangeal joints. When they are paralyzed, the long flexor tendons pull the fingers into a position of flexion, completing the "claw" appearance. If the median nerve were intact, the clawing would be less noticeable in the index and long fingers because the two lumbricals would still be capable of some degree of extension of those interphalangeal joints. If the median nerve alone is injured in the carpal tunnel, there would be loss of thenar opposition but not clawing. If the median and ulnar nerves are both transected at the elbow, the hand appears totally flat because of the loss of long flexors, in addition to intrinsic paralysis.

A 15-year-old boy received a shotgun wound to the ventral surface of the upper limb. Three months after the injury the patient exhibits a complete claw hand but can extend his wrist. What is the nature of this patient's injury? A. The ulnar nerve has been severed at the wrist. B. The median nerve has been injured in the carpal tunnel. C. The median and ulnar nerves are damaged at the wrist. D. The median and ulnar nerves have been injured at the elbow region. E. The median, ulnar, and radial nerves have been injured at midhumerus.

D. The musculocutaneous nerve innervates the brachialis and biceps brachii muscles, which are the main flexors at the elbow. The biceps brachii inserts on the radius and is an important supinator. Because the musculocutaneous nerve is damaged in this case, it leads to loss of sensory perception to the lateral forearm, which is supplied by the distal continuation of the musculocutaneous nerve (known as the lateral antebrachial cutaneous nerve). The name "musculocutaneuous" indicates it is "muscular" in the arm and "cutaneous" in the forearm. Adduction and abduction of the fingers are mediated by the ulnar nerve and would not be affected in this instance. The flexor pollicis brevis muscle flexes the thumb and is mainly innervated by the recurrent branch of the median nerve. Flexion of the fingers is performed by the long flexors of the fingers and lumbrical muscles, innervated by the median and ulnar nerves. Sensory innervation of the medial forearm is provided by the medial antebrachial cutaneous nerve, usually a direct branch of the medial cord of the brachial plexus.

A 17-year-old boy has weakness of elbow flexion and supination of the left hand after sustaining a knife wound in that arm in a street fight. Examination in the emergency department indicates that a nerve has been severed. Which of the following conditions will also most likely be seen during physical examination? A. Inability to adduct and abduct his fingers B. Inability to flex his fingers C. Inability to flex his thumb D. Sensory loss over the lateral surface of his forearm E. Sensory loss over the medial surface of his forearm

E. The scaphoid (or the older term, navicular) bone is the most commonly fractured carpal bone.

A 17-year-old boy suffered the most common of fractures of the carpal bones when he fell on his outstretched hand. Which bone would this be? A. Trapezium B. Lunate C. Pisiform D. Hamate E. Scaphoid

D. In a lesion of the lower trunk of the brachial plexus, or the C8 and T1 ventral rami, there is sensory loss on the medial forearm and the medial side of hand (dorsal and ventral). The medial cord is an extension of the lower trunk. The medial cord gives origin to the medial antebrachial cutaneous nerve, which supplies the T1 dermatome of the medial side of the antebrachium. The lower lateral brachial cutaneous nerve arises from the radial nerve, C5 and C6. The musculocutaneous nerve arises from the lateral cord, ending in the lateral antebrachial cutaneous nerve, with C5 and C6 dermatome fibers. The intercostobrachial nerve is the lateral cutaneous branch of the T2 ventral primary ramus and supplies skin on the medial side of the arm. The median nerve distributes C6 and C7 sensory fibers to the lateral part of the palm, thumb, index, long finger, and half of the ring finger.

A 19-year-old man fell from a cliff when he was hiking in the mountains. He broke his fall by grasping a tree branch, but he suffered injury to the C8 to T1 spinal nerve ventral rami. Sensory tests would thereafter confirm the nature of his neurologic injury by the sensory loss in the part of the limb supplied by which of the following? A. Lower lateral brachial cutaneous nerve B. Musculocutaneous nerve C. Intercostobrachial nerve D. Medial antebrachial cutaneous nerve E. Median nerve

D. Injury to the deep branch of the ulnar nerve results in paralysis of all interosseous muscles and the lumbrical muscles of digits 4 and 5. Extension of the metacarpophalangeal joints is intact, a function of the radial nerve. Interphalangeal extension of digits 4 and 5 is absent, due to the loss of all interosseous muscle and the lumbricals of digits 4 and 5. Some weak interphalangeal joint extension is still present in digits 2 and 3 because the lumbricals of these two fingers are innervated by the median nerve. The radial nerve and the median nerve appear to be intact in this case. If the ulnar nerve were injured in the midforearm region, there would be sensory loss in the palm and digits 4 and 5 and on the dorsum of the hand. The recurrent branch of the median nerve supplies the thenar muscles; it does not supply lumbricals. Moreover, paralysis of this nerve would have no effect on the interphalangeal joints

A 19-year-old man had suffered a deep laceration to an upper limb when he stumbled and fell on a broken bottle. On examination of hand function it is observed that he is able to extend the metacarpophalangeal joints of all his fingers in the affected limb. He cannot extend the interphalangeal (IP) joints of the fourth and fifth digits, and extension of the IP joints of the second and third digits is very weak. There is no apparent sensory deficit in the hand. Which of the following nerves has most likely been injured? A. Radial nerve at the elbow B. Median nerve at the wrist C. Ulnar nerve in midforearm D. Deep branch of ulnar nerve E. Recurrent branch of the median nerve

D. The supraspinatus muscle is one of the four rotator cuff muscles—the other three being the infraspinatus, teres minor, and subscapularis muscles. The tendon of the supraspinatus muscle is relatively avascular and is often injured when the shoulder is dislocated. This muscle initiates abduction of the arm, and damage would impair this movement. The coracobrachialis muscle, which runs from the coracoid process to the humerus, functions in adduction and flexion of the arm. The main function of the triceps brachii muscle is to extend the elbow, and damage to its long head would not affect abduction. The pectoralis minor muscle functions as an accessory respiratory muscle and to stabilize the scapula and is not involved in abduction. The teres major muscle functions to adduct and medially rotate the arm.

A 19-year-old man is brought to the emergency department after dislocating his shoulder while playing soccer. Following reduction of the dislocation, he has pain over the dorsal region of the shoulder and cannot abduct the arm normally. An MRI of the shoulder shows a torn muscle. Which of the following muscles is most likely to have been damaged by this injury? A. Coracobrachialis B. Long head of the triceps brachii C. Pectoralis minor D. Supraspinatus E. Teres major

A. The flexor carpi ulnaris muscle is not innervated by the radial nerve but rather by the ulnar nerve. The brachioradialis, extensor carpi radialis longus and brevis, and supinator muscles are all innervated by the radial nerve distal to the spiral groove.

A 21-year-old female softball pitcher is examined in the emergency department after she was struck in the arm by a line drive. Plain radiographic and MRI studies show soft tissue injury to the region of the spiral groove, with trauma to the radial nerve. Which of the following muscles would be intact after this injury? A. Flexor carpi ulnaris B. Extensor indicis C. Brachioradialis D. Extensor carpi radialis longus E. Supinator

A. The nail was fired explosively from the nail gun and then pierced the ulnar nerve near the coronoid process of the ulna and the trochlea of the humerus. Paralysis of the medial half of the flexor digitorum profundus muscle would result (among other significant deficits), with loss of flexion of the distal interphalangeal joints of digits 4 and 5. Ulnar trauma at the wrist would not affect the interphalangeal joints, although it would cause paralysis of interossei, hypothenar muscles, and so on. Median nerve damage proximal to the pronator teres would affect proximal interphalangeal joint flexion and distal interphalangeal joint flexion of digits 2 and 3 as well as thumb flexion. Median nerve injury at the wrist would cause loss of thenar muscles but not long flexors of the fingers. Trauma to spinal nerve ventral ramus C8 would affect all long finger flexors.

A 22-year-old male construction worker is admitted to the hospital after he suffers a penetrating injury to his upper limb from a nail gun. Upon physical examination, the patient is unable to flex the distal interphalangeal joints of digits 4 and 5. What is the most likely cause of his injury? A. Trauma to the ulnar nerve near the trochlea B. Trauma to the ulnar nerve at the wrist C. Median nerve damage proximal to the pronator teres D. Median nerve damage at the wrist E. Trauma to spinal nerve root C8

B. As the radial artery passes from the ventral surface of the wrist to the dorsum, it crosses through the anatomic snuffbox, passing over the scaphoid bone. The ulnar artery at the wrist is located on the medial side of the wrist, passing from beneath the flexor carpi ulnaris to reach Guyon's canal between the pisiform bone and the flexor retinaculum. Guyon's canal is adjacent to but not in communication with the carpal tunnel. The anterior interosseous and posterior interosseous arteries arise from the common interosseous branch of the ulnar artery and pass proximal to distal in the forearm between the radius and ulna, in the flexor and extensor compartments, respectively. The deep palmar branch of the ulnar artery passes between the two heads of the adductor pollicis to anastomose with the radial artery in the palm

A 22-year-old male football player suffered a wrist injury while falling with force on his outstretched hand. When the anatomic snuffbox is exposed in surgery, which artery is visualized crossing the fractured bone that provides a floor for this space? A. Ulnar B. Radial C. Anterior interosseous D. Posterior interosseous E. Deep palmar arch

B. The student had broken the neck of the fifth metacarpal when hitting the machine with his fist. This is the more common type of "boxer's fracture." Neither a fracture of the ulnar styloid nor a Colles' fracture nor Smith's fracture of the distal radius would present with the absence of a knuckle as observed here. Bennett's fracture involves dislocation of the carpometacarpal joint of the thumb. Indications are that the injury is on the medial side of the hand, not the wrist, nor the lateral side of the hand or wrist.

A 22-year-old male medical student was seen in the emergency department with a complaint of pain in his right hand. He confessed that he had hit a vending machine in the hospital when he did not receive his soft drink after inserting money twice. The medial side of the dorsum of the hand was quite swollen, and one of his knuckles could not be seen when he "made a fist." The physician made a diagnosis of a "boxer's fracture." What was the nature of the impatient student's injury? A. Fracture of the styloid process of the ulna B. Fracture of the neck of the fifth metacarpal C. Colles' fracture of the radius D. Smith's fracture of the radius E. Bennett's fracture of the thumb

Answer D. Lymph from the skin of the anterior chest wall primarily drains to the axillary lymph nodes.

A 22-year-old man is diagnosed with metastatic malignant melanoma of the skin over the xiphoid process. Which lymph nodes receive most of the lymph from this area and are therefore most likely to be involved in metastasis of the tumor? A. Deep inguinal B. Vertical group of superficial inguinal C. Horizontal group of superficial inguinal D. Axillary E. Deep and superficial inguinal

C. The contraction of the extensor mechanism produces extension of the distal interphalangeal joint. When it is torn from the distal phalanx, the digit is pulled into flexion by the flexor digitorum profundus muscle. If a piece of the distal phalanx is attached to the torn tendon it is an avulsion fracture. The proper palmar digital branches of the median nerve supply lumbrical muscles and carry sensation from their respective digits. Vincula longa are slender, bandlike connections from the deep flexor tendons to the phalanx that can carry blood supply to the tendons. The insertions of the flexor digitorum superficialis and profundus are on the flexor surface of the middle and distal phalanges, respectively, and act to flex the interphalangeal joints.

A 23-year-old female maid was making a bed in a hotel bedroom. As she straightened the sheet by running her right hand over the surface with her fingers extended, she caught the end of the index finger in a fold. She experienced a sudden, severe pain over the base of the terminal phalanx. Several hours later when the pain had diminished, she noted that the end of her right index finger was swollen and she could not completely extend the terminal interphalangeal joint. Which one of the following structures within the digit was most likely injured? A. The proper palmar digital branch of the median nerve B. The vinculum longa C. The insertion of the tendon of the extensor digitorum onto the base of the distal phalanx D. The insertion of the flexor digitorum profundus tendon E. The insertion of the flexor digitorum superficialis tendon

D. The recurrent branch of the median nerve is motor to the muscles of the thenar eminence, which is an elevation caused by the abductor pollicis brevis, flexor pollicis brevis, and opponens pollicis muscles. If the opponens pollicis is paralyzed, one cannot oppose the pad of the thumb to the pads of the other digits because this is the only muscle that can oppose the thumb by moving the first metacarpal on the trapezium. The recurrent branch does not have a cutaneous distribution. Holding a piece of paper between the fingers is a simple test of adduction of the fingers. These movements are controlled by the deep branch of the ulnar nerve, which is not injured in this patient.

A 24-year-old man is admitted with a wound to the palm of his hand. He cannot touch the pad of his index finger with his thumb but can grip a sheet of paper between all fingers and has no loss of sensation on the skin of his hand. Which of the following nerves has most likely been injured? A. Deep branch of ulnar B. Anterior interosseous C. Median D. Recurrent branch of median E. Deep branch of radial

D. The acromioclavicular ligament connects the clavicle to the coracoid process of the scapula. Separation of the shoulder (dislocation of the acromioclavicular [AC] joint) is associated with damage to the acromioclavicular ligament (capsule of the AC joint) and, in more severe injuries, disruption of the coracoclavicular ligaments (conoid and trapezoid portions). The glenohumeral ligament may be injured by an anterior dislocation of the humerus but is not likely to be injured by a separated shoulder. The coracoacromial ligament, transverse scapular ligament, and tendon of the long head of triceps brachii are not likely to be injured by separation of the shoulder.

A 23-year-old male basketball player is admitted to the hospital after injuring his shoulder during a game. Physical and radiographic examinations reveal total separation of the shoulder (Fig. 6-5). Which of the following structures has most likely been torn? A. Glenohumeral ligament B. Coracoacromial ligament C. Tendon of long head of biceps brachii D. Acromioclavicular ligament E. Transverse scapular ligament

C. The radial nerve is the most likely nerve compressed to cause these symptoms. This type of nerve palsy is often called "Saturday night palsy." One reason for this nickname is that people would supposedly fall asleep after being intoxicated on a Saturday night with their arm over the back of a chair or bench, thereby compressing the nerve in the spiral groove. The radial nerve innervates all of the extensors of the elbow, wrist, and fingers. It innervates the triceps brachii muscle but the motor branch typically comes off proximal to the site of compression, so the patient can still extend the elbow. Paralysis of the lateral cord of the brachial plexus would result in loss of the musculocutaneous nerve and the pectoral nerves, which do not mediate extension of the forearm or hand. The medial cord of the brachial plexus branches into the median nerve and ulnar nerve. Neither of these nerves innervates muscles that control extension. The median nerve innervates flexors of the forearm and the thenar muscles. The lateral and median pectoral nerves do not extend into the arm and innervate the pectoralis major and minor muscles.

A 23-year-old male medical student fell asleep in his chair with Netter's Atlas wedged into his axilla. When he awoke in the morning, he was unable to extend his wrist or fingers. Movements of the ipsilateral shoulder joint appear to be normal. Which of the following nerves was most likely compressed, producing the symptoms described? A. Lateral cord of the brachial plexus B. Medial cord of the brachial plexus C. Radial nerve D. Median nerve E. Lateral and medial pectoral nerves

Answer C. Tenosynovitis can be due to an infection of the synovial sheaths of the digits. Tenosynovitis in the thumb may spread through the synovial sheath of the flexor pollicis longus tendon, also known as the radial bursa. The tendons of the flexor digitorum superficialis and profundus muscles are enveloped in the common synovial flexor sheath or ulnar bursa. Neither the flexor carpi radialis nor flexor pollicis brevis tendons are contained in synovial flexor sheaths.

A 24-year-old female medical student was bitten at the base of her thumb by her dog. The wound became infected and the infection spread into the radial bursa. The tendon(s) of which muscle will most likely be affected? A. Flexor digitorum profundus B. Flexor digitorum superficialis C. Flexor pollicis longus D. Flexor carpi radialis E. Flexor pollicis brevis

A. The nerve most likely affected is the radial nerve which innervates the triceps brachii muscle. The axillary, median and ulnar nerves do not innervate any muscles in the arm. The musculocutaneous nerve is innervating the coracobrachialis, biceps brachii, and brachialis muscles in the anterior compartment of the arm.

A 25-year-old man falls on a slippery trail and injures his upper limb. Inspection reveals abrasions over his arm at the area of the triceps muscle. Which of the following nerves innervates the triceps muscle? A. Radial B. Axillary C. Median D. Ulnar E. Musculocutaneous

A.The nerve most likely affected in the radiograph is the radial nerve as it travels in the radial groove along the humerus as it descends to the forearm. The radial nerve innervates the triceps brachii muscle (and extensors in the forearm), whereas the biceps brachii, coracobrachialis, and brachialis muscles are innervated by the musculocutaneous nerve and the deltoid muscle by the axillary nerve.

A 25-year-old man falls on a slippery trail and injures his upper limb. Inspection reveals abrasions over his arm. A radiograph shows a fracture at the radial groove of the humerus. Which of the following muscles will most likely be paralyzed? A. Triceps brachii B. Biceps brachii C. Coracobrachialis D. Brachialis E. Deltoid

A. The nerve likely affected in the radiograph is the radial nerve as it travels in the radial groove as it descends along the humerus to the forearm. The radial nerve innervates the extensor muscles of the forearm. Once these muscles have lost their inability to contract because the radial nerve is injured, the wrist cannot be extended. This is referred to as wrist drop. B: Inability to flex the hand and loss of pronation is an unlikely choice as flexion of the hand is performed via innervation of muscles by the median and ulnar nerves, pronation of the hand is also performed by muscles innervated by the median nerve. C: Inability to flex index and middle fingers at the proximal and distal interphalangeal joints is a function of the median nerve. D: Inability to flex the ring and little fingers at the distal interphalangeal joints describes an ulnar nerve branch deficit. E: Paralysis of lumbricals and interosseous muscles is an incorrect choice as these muscles are innervated by branches of the median and ulnar nerve which was likely spared during the injury shown on the radiograph

A 25-year-old man falls on a slippery trail and injures his upper limb. Inspection reveals abrasions over his arm. A radiograph shows a fracture at the radial groove of the humerus. Which of the following will be present during physical examination? A. Wrist drop B. Inability to flex his hand and loss of pronation C. Inability to flex index and middle fingers at the distal interphalangeal (DIP) and proximal interphalangeal (PIP) joints D. Inability to flex ring and little fingers at DIP E. Paralysis of lumbricals and interosseous muscles

A. The nerve likely affected in the radiograph is the radial nerve as it travels in the radial groove as it descends along the humerus to the forearm. Although the radial nerve does not provide innervations to muscles that close the hand, it does innervate extensor carpi radialis longus and brevis muscles via the posterior interosseous nerve. These muscles act as wrist extensors and are necessary for making a fist, as they contract synergistically with the flexors of the fingers, making it indispensable while performing this action. The "position of function" of the hand is with the wrist extended about 30 degrees. B: Inability to flex the hand and loss of pronation is an unlikely choice as flexion of the hand is performed via innervation of muscles by the median and ulnar nerves, pronation of the hand is also performed by a muscle innervated by the median nerve. C: Inability to flex index and middle fingers at the proximal and distal interphalangeal joints is a function of the median nerve. D: Inability to flex ring and little fingers at the distal interphalangeal describes ulnar nerve branch deficit. E: Paralysis of lumbricals and interosseous muscles is an incorrect choice as these muscles are innervated by branches of the median and ulnar nerves, which were likely spared during the injury shown on the radiograph

A 25-year-old man falls on a slippery trail and injures his upper limb. Inspection reveals abrasions over his arm. The radiograph of his arm is shown in Fig. 6-1. Which deficit will most likely be encountered? A. Inability to make a fist B. Inability to flex his hand and loss of pronation C. Inability to flex index and middle fingers at DIP and PIP joints D. Inability to flex ring and little fingers at DIP E. Paralysis of lumbricals and interosseous muscles

A. The long head of the biceps brachii muscle assists in shoulder flexion and during a tendinopathy would cause pain in the anterior compartment of the shoulder, where it originates at the supraglenoid tubercle. Also, forced contraction would cause a greater tension force on the tendon.

A 26-year-old male power lifter visits the outpatient clinic with a painful shoulder. Radiographic examination reveals tendinopathy of the long head of the biceps brachii muscle. Which of the following conditions will most likely be present during physical examination? A. Pain is felt in the anterior shoulder during forced contraction B. Pain is felt in the lateral shoulder during forced contraction C. Pain is felt during abduction and flexion of the shoulder joint D. Pain is felt during extension and adduction of the shoulder joint E. Pain is felt in the lateral shoulder during flexion of the shoulder joint

C. Fracture of the surgical neck of the humerus often injures the axillary nerve, which innervates the deltoid and teres minor muscles. Abduction of the humerus between 15 degrees and the horizontal is performed by the deltoid muscle. Lateral rotation of the humerus is mainly performed by the deltoid muscle, teres minor, and the infraspinatus. The deltoid and teres minor are both lost in this case. Fracture of the glenoid fossa would lead to drooping of the shoulder. Fracture of the anatomic neck of the humerus will similarly lead to a drooping of the shoulder but would not necessarily affect abduction of the humerus. It is also quite unusual. Fracture of the middle third of the humerus would most likely injure the radial nerve. The ulnar nerve would be potentially compromised in a fracture of the medial epicondyle of the humerus.

A 27-year-old male painter is admitted to the hospital after falling from a ladder. Physical examination reveals that the patient is unable to abduct his arm more than 15 degrees and he cannot rotate the arm laterally. A radiographic examination reveals an oblique fracture of the humerus. He has associated sensory loss over the shoulder area. Which of the following injuries will most likely correspond to the symptoms of the physical examination? A. Fracture of the medial epicondyle B. Fracture of the glenoid fossa C. Fracture of the surgical neck of the humerus D. Fracture of the anatomic neck of the humerus E. Fracture of the middle third of the humerus

E. The surgeon took the distal segments of the median nerves from both forearms, mistakenly believing them to be palmaris longus tendons. Both of the structures lie in the midline of the ventral surface of the distal forearm and are often of similar appearance in color and diameter. The nerve is located deep to the tendon, when the tendon is present, but when the tendon is absent, the nerve appears to be where the tendon belongs. There is no evidence of rib fractures; even so, a fractured rib would not explain loss of sensation on the lateral portion of the palm. Lower plexus trauma (C8, T1) would result in paralysis of forearm flexor muscles and all intrinsic hand muscles and sensory loss over the medial dorsum of the hand, in addition to palmar sensory loss. Dupuytren's contracture is a flexion contracture of (usually) digits four and five from connective tissue disease in the palm. Radial nerve injury in the posterior forearm would affect metacarpophalangeal joint extension, thumb extension, and so on, not palmar disturbances.

A 27-year-old man had lost much of the soft tissue on the dorsum of his left hand in a motorcycle crash. Imaging studies show no other upper limb injuries. Because function of the left extensor carpi radialis longus and brevis tendons was lost, it was decided to replace those tendons with the palmaris longus tendons from both forearms because of their convenient location and relative unimportance. Postoperatively, it is found that sensation is absent in both hands on the lateral palm and palmar surfaces of the first three digits; there is also paralysis of thumb opposition. What is the most likely cause of the sensory deficit and motor loss in both thumbs? A. Bilateral loss of spinal nerve T1 with fractures of first rib bilaterally B. Lower plexus (lower trunk) trauma C. Dupuytren contracture D. Left radial nerve injury in the posterior compartment of the forearm E. The palmaris longus was absent bilaterally; the nerve normally beneath it looked like a tendon and was cut

Answer A. The lower subscapular nerve arises from the cervical spinal nerves 5 and 6. It innervates the subscapularis and teres major muscles. The subscapularis and teres major muscles are both responsible for adducting and medially rotating the arm. A lesion of this nerve would result in weakness in these motions. The axillary nerve also arises from cervical spinal nerves 5 and 6 and innervates the deltoid and teres minor muscles. The deltoid muscle is large and covers the entire surface of the shoulder, and contributes to arm movement in any plane. The teres minor muscle is a lateral rotator and a member of the rotator cuff group of muscles. The radial nerve arises from the posterior cord of the brachial plexus. It is the largest branch, and it innervates the triceps brachii and anconeus muscles in the arm. The spinal accessory nerve is cranial nerve XI, and innervates the trapezius muscle, which elevates and depresses the scapula. The ulnar nerve arises from the medial cord of the brachial plexus and runs down the medial aspect of the arm. It innervates muscles of the forearm and hand.

A 27-year-old man was admitted to the emergency department after an automobile collision in which he suffered a fracture of the lateral border of the scapula. Six weeks after the accident, physical examination reveals weakness in medial rotation and adduction of the humerus. Which nerve was most likely injured? A. Lower subscapular B. Axillary C. Radial D. Spinal accessory E. Ulnar

A. The patient exhibits the classic "benediction attitude" of the thumb and fingers from injury to the median nerve proximally in the forearm. The thumb is somewhat extended (radial supplied abductor and extensors unopposed); digits 2 and 3 are extended (by intact interossei); digits 4 and 5 are partially flexed (by their intact flexor digitorum profundus). A lesion of the median nerve would result in weakened flexion of the proximal interphalangeal joints of all digits (flexor digitorum superficialis muscle), loss of flexion of the interphalangeal joint of the thumb, the distal interphalangeal joints of digits 2 and 3 (flexor digitorum profundus muscle), and weakened flexion of the metacarpophalangeal joints of the second and third digits (first and second lumbricals). A lesion of both the ulnar and median nerves would cause weakness or paralysis of flexion of all of the digits. A lesion of the ulnar nerve would mostly cause weakness in flexion of the DIP of the fourth and fifth digits and would affect all of the interosseous muscles and the lumbricals of the third and fourth digits. A lesion of the radial nerve would cause weakness in extension of the wrist, thumb, and metacarpophalangeal joints.

A 29-year-old woman had sustained a deep laceration in the proximal part of the forearm. After the wound is closed, the following functional deficits are observed by the neurologist on service: the first three digits are in a position of extension and cannot be flexed; digits 4 and 5 are partially flexed at the metacarpophalangeal (MCP) joints and noticeably more flexed at the distal interphalangeal joints; sensation is absent in the lateral side of the palm and the palmar surfaces of digits 1 to 3 and half of digit 4. Which of the following nerve(s) has (have) most likely been injured? A. Median nerve B. Ulnar and median nerves C. Ulnar nerve D. Radial and ulnar nerves E. Radial nerve

B. In shoulder separation, either or both the acromioclavicular and coracoclavicular ligaments can be partially or completely torn through. The acromioclavicular joint can be interrupted and the distal end of the clavicle may deviate upward in a complete separation, while the upper limb droops away inferiorly, causing a "step off" that can be palpated and sometimes observed. Displacement of the head of the humerus is shoulder dislocation, not separation. The coracoacromial ligament is not torn in separation (but it is sometimes used in the repair of the torn coracoclavicular ligament). Disruption of the glenoid labrum often accompanies shoulder dislocation.

A 34-year-old female skier was taken by ambulance to the hospital after she struck a tree on the ski slope. Imaging gives evidence of a shoulder separation. Which of the following typically occurs in this kind of injury? A. Displacement of the head of the humerus from the glenoid cavity B. Partial or complete tearing of the coracoclavicular ligament C. Partial or complete tearing of the coracoacromial ligament D. Rupture of the transverse scapular ligament E. Disruption of the glenoid labrum

D. The subclavian vein traverses between the clavicle and first rib and is the most superficial structure to be damaged following a fracture of the clavicle. The subclavian artery runs posterior to the subclavian vein, and though it is in the appropriate location, it would likely not be damaged because of its deep anatomic position. The cephalic vein is a tributary to the axillary vein after ascending on the lateral side of the arm. Its location within the body is too superficial and lateral to the site of injury. The lateral thoracic artery is a branch from the axillary artery that runs lateral to the pectoralis minor. It courses inferior and medial from its point of origin from the axillary artery, and it does not maintain a position near the clavicle during its descent. The internal thoracic artery arises from the first part of the subclavian artery before descending deep to the costal cartilages. Its point of origin from the subclavian artery is lateral to clavicular injury. Furthermore, its course behind the costal cartilages is quite medial to the clavicular fracture.

A 29-year-old woman is examined in the emergency department after falling from her balcony. Radiographic examination reveals that she has suffered a broken clavicle, with associated internal bleeding. Which of the following vessels is most likely to be injured in clavicular fractures? A. Subclavian artery B. Cephalic vein C. Lateral thoracic artery D. Subclavian vein E. Internal thoracic artery

A. Because the median nerve is injured within the cubital fossa, the long flexors are paralyzed, including the flexor pollicis longus muscle. The flexor pollicis longus would not be paralyzed if the median nerve were injured at the wrist. Lateral palm sensory loss confirms median nerve injury. If only the anterior interosseous nerve were damaged, there would be no cutaneous sensory deficit. The radial nerve supplies wrist extensors, long thumb abductor, and metacarpophalangeal joint extensors. The ulnar nerve does not supply sensation to the lateral palm.

A 31-year-old female figure skater is examined in the emergency department following an injury that forced her to withdraw from competition. When her male partner missed catching her properly from an overhead position, he grasped her powerfully, but awkwardly, by the forearm. Clinical examination demonstrated a positive Ochsner test, inability to flex the distal interphalangeal joint of the index finger on clasping the hands. In addition, she is unable to flex the terminal phalanx of the thumb and has loss of sensation over the thenar half of the hand. What is the most likely nature of her injury? A. Median nerve injured within the cubital fossa B. Anterior interosseous nerve injury at the pronator teres C. Radial nerve injury at its entrance into the posterior forearm compartment D. Median nerve injury at the proximal skin crease of the wrist E. Ulnar nerve trauma halfway along the forearm

A. The quadrangular space is bordered medially by the long head of the triceps brachii muscle, laterally by the surgical neck of the humerus, superiorly by the teres minor and subscapularis muscles, and inferiorly by the teres major muscle. Both the axillary nerve and posterior circumflex humeral vessels traverse this space. The other structures listed are not contained within the quadrangular space. The cephalic vein is located in the deltopectoral triangle, and the radial nerve is located in the triangular interval.

A 35-year-old male body builder has enlarged his shoulder muscles to such a degree that the size of the quadrangular space is greatly reduced. Which of the following structures would most likely be compressed in this condition? A. Axillary nerve B. Anterior circumflex humeral artery C. Cephalic vein D. Radial nerve E. Subscapular artery

C. The surgical neck of the humerus is a typical site of fractures. The fracture line lies above the insertions of the pectoralis major, teres major, and latissimus dorsi muscles. The supraspinatus muscle abducts the proximal fragment, whereas the distal fragment is elevated and adducted. The elevation results from contraction of the deltoid, biceps brachii, and coracobrachialis muscles. The adduction is due to the action of pectoralis major, teres major, and latissimus dorsi.

A 31-year-old male hockey player fell on his elbow and is admitted to the emergency department. Radiographic examination reveals a fracture of the surgical neck of the humerus, producing an elevation and adduction of the distal fragment. Which of the following muscles would most likely cause the adduction of the distal fragment? A. Brachialis B. Teres minor C. Pectoralis major D. Supraspinatus E. Pectoralis minor

Answer B. The musculocutaneous nerve supplies the biceps brachii and brachialis muscles, which are the flexors of the forearm at the elbow. The musculocutaneous nerve continues as the lateral antebrachial cutaneous nerve, which supplies sensation to the lateral side of the forearm (with the forearm in the anatomic position). The biceps brachii muscle is the most powerful supinator muscle. Injury to this nerve would result in weakness of supination and forearm flexion and lateral forearm sensory loss. Injury to the radial nerve would result in weakened extension and a characteristic wrist drop. Injury to the median nerve causes paralysis of flexor digitorum superficialis muscle and other flexors in the forearm and results in a characteristic flattening of the thenar eminence. The lateral cord of the brachial plexus gives origin both to the musculocutaneous and lateral pectoral nerves. There is no indication of pectoral paralysis or weakness. Injury to the lateral cord can result in weakened flexion and supination in the forearm, and weakened adduction and medial rotation of the arm. The lateral cutaneous nerve of the forearm is a branch of the musculocutaneous nerve and does not supply any motor innervation. Injury to the musculocutaneous nerve alone is unusual but can follow penetrating injuries.

A 32-year-old woman is admitted to the emergency department after an automobile collision. Radiologic examination reveals multiple fractures of the humerus. Flexion and supination of the forearm are severely weakened. She also has loss of sensation on the lateral surface of the forearm. Which of the following nerves has most likely been injured? A. Radial B. Musculocutaneous C. Median D. Lateral cord of brachial plexus E. Lateral cutaneous nerve of the forearm

C. The head of the humerus is displaced inferiorly because in that location it is not supported by rotator cuff muscle tendons or the coracoacromial arch. It is also pulled anteriorly (relative to the tendon of the triceps brachii) beneath the coracoid process by pectoralis and subscapularis muscles. It would not be displaced posteriorly because it is supported by the teres minor and infraspinatus muscle tendons. It would not be displaced superiorly because the coracoacromial ligament and supraspinatus reinforce in that direction. A medial dislocation is blocked by the subscapularis tendon.

A 35-year-old male wrestler is admitted to the emergency department with excruciating pain in his right shoulder and proximal arm. During physical examination, the patient clutches the arm at the elbow with his contralateral hand and is unable to move the injured limb. Radiographic studies show that the patient has a dislocation of the humerus at the glenohumeral joint. Which of the following conditions is the most likely? A. The head of the humerus is displaced anteriorly B. The head of the humerus is displaced posteriorly C. The head of the humerus is displaced inferiorly D. The head of the humerus is displaced superiorly E. The head of the humerus is displaced medially

C. The common palmar digital branch comes off the superficial branch of the ulnar nerve and supplies the skin of the little finger and the medial side of the ring finger. The superficial branch of the radial nerve provides cutaneous innervation to the radial (lateral) dorsum of the hand and the radial two and a half digits over the proximal phalanx. The common palmar digital branch of the median nerve innervates most of the lateral aspect of the palmar hand and the dorsal aspect of the second and third finger as well as the lateral part of the fourth digit. The deep radial nerve supplies the extensor carpi radialis brevis and supinator muscles and continues as the posterior interosseous nerve. The recurrent branch of the median nerve supplies the abductor pollicis brevis, flexor pollicis brevis, and opponens pollicis muscles.

A 35-year-old man has a small but painful tumor under the nail of his little finger. Which of the following nerves would have to be anesthetized for a painless removal of the tumor? A. Superficial radial B. Common palmar digital of median C. Common palmar digital of ulnar D. Deep radial E. Recurrent branch of median

E. The ulnar nerve innervates the palmar interossei, which adduct the fingers. This is the movement that would maintain the paper between the fingers. The axillary nerve does not innervate muscles of the hand. The median nerve supplies the first and second lumbricals, the opponens pollicis, abductor pollicis brevis, and the flexor pollicis brevis muscles. None of these muscles would affect the ability to hold a piece of paper between the fingers. The musculocutaneous and radial nerves do not supply muscles of the hand.

A 36-year-old man is brought to the emergency department because of a deep knife wound on the medial side of his distal forearm. He is unable to hold a piece of paper between his fingers and has sensory loss on the medial side of his hand and little finger. Which nerve is most likely injured? A. Axillary B. Median C. Musculocutaneous D. Radial E. Ulnar

A. The circumflex scapular artery passes through the triangular space after arising from the subscapular artery. It provides superficial branches to the overlying latissimus dorsi, whereas its deep portion passes into the infraspinous fossa to anastomose with the suprascapular artery. The dorsal scapular artery passes between the ventral rami of the brachial plexus and then deep to the medial border of the scapula. The transverse cervical artery arises from the thyrocervical trunk at the root of the neck and can provide origin for a dorsal scapular branch. The lateral thoracic and thoracoacromial arteries are branches of the second part of the axillary artery and provide no supply to the latissimus dorsi.

A 41-year-old woman is scheduled for a latissimus dorsi muscle flap to cosmetically augment the site of her absent left breast after mastectomy. Part of the latissimus dorsi muscle is advanced to the anterior thoracic wall, based upon arterial supply provided in part by the artery that passes through the triangular space of the axilla. Which artery forms the vascular base of this flap? A. Circumflex scapular artery B. Dorsal scapular artery C. Transverse cervical artery D. Lateral thoracic artery E. Thoracoacromial artery

C. The common extensor tendon originates from the lateral epicondyle, and inflammation of this tendon is lateral epicondylitis, nicknamed "tennis elbow" because the tendon is often irritated during the backhand stroke in tennis. Because the extensors of the wrist originate as part of the common extensor tendon, extension of the wrist will exacerbate the pain of lateral epicondylitis.

A 43-year-old female tennis player visits the outpatient clinic with pain over the right lateral epicondyle of her elbow. Physical examination reveals that the patient has lateral epicondylitis. Which of the following tests should be performed during physical examination to confirm the diagnosis? A. Nerve conduction studies B. Evaluation of pain experienced during flexion and extension of the elbow joint C. Observing the presence of pain when the wrist is extended against resistance D. Observing the presence of numbness and tingling in the ring and little fingers when the wrist is flexed against resistance E. Evaluation of pain felt over the styloid process of radius during brachioradialis contraction

C. C7 is the main spinal nerve that contributes to the radial nerve and innervates the triceps brachii. Absence of the triceps reflex is usually indicative of a C7 radiculopathy or injury.

A 43-year-old man is admitted to the hospital, having suffered a whiplash injury when his compact automobile was struck from behind by a sports utility vehicle. MRI examination reveals some herniation of a disc in the cervical region. Physical examination reveals that the patient has lost elbow extension; there is absence of his triceps reflex and loss of extension of the metacarpophalangeal joints on the ipsilateral side. Which of the following spinal nerves is most likely affected? A. C5 B. C6 C. C7 D. C8 E. T1

B. Froment's sign is positive for ulnar nerve palsy. More specifically it tests the action of the adductor pollicis muscle. The patient is asked to hold a sheet of paper between the thumb and a flat palm. The flexor pollicis longus is innervated by the anterior interosseous branch of the median nerve. The flexor digiti minimi is innervated by the deep branch of the ulnar nerve and would not be used to hold a sheet of paper between the thumb and palm. The flexor carpi radialis is innervated by the median nerve, and the extensor indicis is innervated by the radial nerve

A 43-year-old woman visits the outpatient clinic with a neurologic problem. Diagnostically, she cannot hold a piece of paper between her thumb and the lateral side of her index finger without flexing the distal joint of her thumb. This is a positive Froment's sign, which is consistent with ulnar neuropathy. Weakness of which specific muscle causes this sign to appear? A. Flexor pollicis longus B. Adductor pollicis C. Flexor digiti minimi D. Flexor carpi radialis E. Extensor indicis

C. Inability to extend the metacarpophalangeal (MCP) joints. The tendons of the extensor digitorum and extensor digiti minimi muscles, innervated by the radial nerve, are responsible for extension of the MCP and, to a much lesser degree, the proximal (PIP) and distal (DIP) interphalangeal joints. Abduction and adduction of the MCP joints are functions of the interossei, all of which are innervated by the deep ulnar nerve. Extension of the PIP and DIP joints is performed by the lumbricals and interossei. The first two lumbricals are supplied by the median nerve; the other lumbricals and the interossei, by the deep branch of the ulnar nerve.

A 44-year-old woman is diagnosed with radial nerve palsy. When muscle function is examined at the metacarpophalangeal (MCP), proximal interphalangeal (PIP), and distal interphalangeal (DIP) joints, what findings are most likely to be present? A. Inability to abduct the digits at the MCP joint B. Inability to adduct the digits at the MCP joint C. Inability to extend the MCP joints only D. Inability to extend the MCP, PIP, and DIP joints E. Inability to extend the PIP and DIP joints

C. A fracture of the humerus just proximal to the epicondyles is called a supracondylar fracture. This is the most common cause of a Volkmann ischemic fracture. The sharp bony fragment often lacerates the brachial (or other) artery, with bleeding into the flexor compartment. Diminution of arterial supply to the compartment results in the ischemia. Bleeding into the compartment causes greatly increased pressure, first blocking venous outflow from the compartment, then reducing the arterial flow into the compartment as the pressure rises to arterial levels. The ischemic muscles then undergo unrelieved contracture. A humeral fracture is sometimes placed in a cast from shoulder to wrist, often concealing the ischemia until major tissue loss occurs. Cold, insensate digits, and great pain are warnings of this compartmental syndrome, demanding that the cast be removed and the compartment opened ("released") for pressure reduction and vascular repair. Fracture of the surgical neck endangers the axillary nerve and posterior circumflex humeral artery, although not ischemic contracture. Fracture of the humerus in the spiral groove can injure the radial nerve and profunda brachii artery. Fracture of the olecranon does not result in Volkmann's contracture, although the triceps brachii can displace the distal fractured fragment of the ulna.

A 45-year-old female motorcyclist, propelled over the handlebars of her bike by an encounter with a rut in the road, lands on the point of one shoulder. She is taken by ambulance to the emergency department. During physical examination, the arm appears swollen, pale, and cool. Any movement of the arm causes severe pain. Radiographic examination reveals a fracture and a large hematoma, leading to diagnosis of Volkmann's ischemic contracture. At which of the following locations has the fracture most likely occurred? A. Surgical neck of humerus B. Radial groove of humerus C. Supracondylar line of humerus D. Olecranon E. Lateral epicondyle

Answer D. Fracture of the medial epicondyle often causes damage to the ulnar nerve due to its position in the groove behind the epicondyle. The ulnar nerve innervates one and a half muscles in the forearm, the flexor carpi ulnaris and the medial half of the flexor digitorum profundus muscles. The nerve continues on to innervate most of the muscles in the hand. The flexor digitorum superficialis is innervated by the median nerve and the biceps brachii muscle by the musculocutaneous. The radial nerve innervates both the brachioradialis and supinator muscles.

A 45-year-old man arrived at the emergency department with injuries to his left elbow after he fell in a bicycle race. Plain radiographic and magnetic resonance imaging (MRI) examinations show a fracture of the medial epicondyle and an injured ulnar nerve. Which of the following muscles will most likely be paralyzed? A. Flexor digitorum superficialis B. Biceps brachii C. Brachioradialis D. Flexor carpi ulnaris E. Supinator

E. In Smith's fracture, the distal fragment of the radius deviates palmarward, often displacing the lunate bone. The other listed bones are unlikely to be displaced in a palmar direction by Smith's fracture.

A 45-year-old man had fallen on his outstretched hand, resulting in Smith's fracture of the distal end of the radius. The fractured bone displaced a carpal bone in the palmar direction, resulting in nerve compression within the carpal tunnel. Which of the following carpal bones will most likely be dislocated? A. Scaphoid B. Trapezium C. Capitate D. Hamate E. Lunate

A. The medial antebrachial cutaneous nerve carries sensory fibers derived from the C8 and T1 levels. The lateral antebrachial cutaneous nerve is the distal continuation of the musculocutaneous nerve, carrying fibers from the C5, C6, and C7 levels. The deep branch of the ulnar nerve and the anterior interosseous nerves carry predominantly motor fibers. The sensory fibers coursing in the radial nerve are derived from the C5 to C8 levels.

A 45-year-old man is admitted to the hospital after a car crash. Radiographic examination reveals mild disc herniations of C7, C8, and T1. The patient presents with a sensory deficit of the C8 and T1 spinal nerve dermatomes. The dorsal root ganglia of C8 and T1 would contain cell bodies of sensory fibers carried by which of the following nerves? A. Medial antebrachial cutaneous nerve B. Long thoracic nerve C. Lateral antebrachial cutaneous nerve D. Deep branch of ulnar nerve E. Anterior interosseous nerve

A. The flexor digitorum profundus muscle is dually innervated by the ulnar nerve to the medial two fingers and the median nerve for the long and index fingers. Because of the superficial course of the ulnar nerve, it is vulnerable to laceration. Such an injury would result in an inability to flex the distal interphalangeal joints of the fourth and fifth digits because the flexor digitorum profundus muscle is the only muscle that flexes this joint. The flexor digitorum superficialis muscle is innervated by the median nerve only, and the course of this nerve runs too deep to be usually affected by lacerations. The lumbricals function to flex the MP joints and assist in extending the IP joints. The interossei adduct and abduct the fingers.

A 45-year-old man is admitted to the hospital after accidentally walking through a plate glass door in a bar while intoxicated. Physical examination shows multiple lacerations to the upper limb, with inability to flex the distal interphalangeal joints of the fourth and fifth digits. Which of the following muscles is most likely affected? A. Flexor digitorum profundus B. Flexor digitorum superficialis C. Lumbricals D. Flexor digitorum profundus and flexor digitorum superficialis E. Interossei

B. In boutonnière deformity, the central portion of the extensor tendon expansion is torn over the proximal interphalangeal (PIP) joint, allowing the tendon to move toward the palm, causing the tendon to act as a flexor of the PIP joint. This causes the distal interphalangeal (DIP) joint to be hyperextended. The tear in the extensor tendon is said to resemble a buttonhole (boutonnière in French), and the head of the proximal phalanx may stick through the hole.

A 45-year-old man visits the outpatient clinic after a digit of his left hand was injured when a door was slammed on his hand. A superficial cut on his middle finger has been sutured, but functional deficits are observed in the finger. The proximal interphalangeal joint is pulled into constant flexion, whereas the distal interphalangeal joint is held in a position of hyperextension. What is the most likely diagnosis? A. Mallet finger B. Boutonnière deformity C. Dupuytren's contracture D. Swan-neck deformity E. Silver fork wrist deformity

C. The spinal accessory nerve (CN XI) arises from the ventral rootlets of C1 to C4 that ascend through the foramen magnum to then exit the cranial cavity through the jugular foramen. It innervates the sternocleidomastoid and trapezius muscles, which function in head rotation and raising of the shoulders. The suprascapular nerve receives fibers from C5-6 (occasionally from C4 if the plexus is "prefixed") and innervates the supraspinatus muscle, which is responsible for the first 15 degrees of arm abduction. Erb's point of the brachial plexus is at the union of C5-6 spinal nerves. The long thoracic nerve arises from plexus routes C5, 6, and 7, and supplies the serratus anterior.

A 45-year-old woman is admitted to the hospital with neck pain. A computed tomography (CT) scan reveals a tumor on the left side of her oral cavity. The tumor and related tissues are removed and a radical neck surgical procedure is performed. Two months postoperatively the patient's left shoulder droops quite noticeably. Physical examination reveals distinct weakness in turning her head to the right and impairment of abduction of her left upper limb to the level of the shoulder. Which of the following structures was most likely injured during the radical neck surgery? A. Suprascapular nerve B. Long thoracic nerve C. Spinal accessory nerve D. The junction of spinal nerves C5 and C6 of the brachial plexus E. Radial nerve

C. The seventh cervical nerve makes a major contribution to the radial nerve, and this nerve is the prime mover in wrist extension. The dermatome of C7 is in the region described.

A 45-year-old woman is admitted to the hospital with neck pain. An MRI examination reveals a herniated disc in the cervical region. Physical examination reveals weakness in wrist extension and paraesthesia on the back of her arm and forearm. Which of the following spinal nerves is most likely injured? A. C5 B. C6 C. C7 D. C8 E. T1

E. The coracoacromial ligament contributes to the coracoacromial arch, preventing superior displacement of the head of the humerus. Because this ligament is very strong, it will rarely be damaged; instead, the ligament can cause inflammation or erosion of the tendon of the supraspinatus muscle as the tendon passes back and forth under the ligament. The acromioclavicular ligament, connecting the acromion with the lateral end of the clavicle, is not in contact with the supraspinatus tendon. The coracohumeral ligament is located too far anteriorly to impinge upon the supraspinatus tendon. The glenohumeral ligament is located deep to the rotator cuff muscles and would not contribute to injury of the supraspinatus muscle. The transverse scapular ligament crosses the scapular notch and is not in contact with the supraspinatus tendon.

A 47-year-old female tennis professional is informed by her physician that she has a rotator cuff injury that will require surgery. Her physician explains that over the years of play, a shoulder ligament has gradually caused severe damage to the underlying muscle. To which of the following ligaments is the physician most likely referring? A. Acromioclavicular ligament B. Coracohumeral ligament C. Transverse scapular ligament D. Glenohumeral ligament E. Coracoacromial ligament

B. When cutaneous lymphatics of the breast are blocked by cancer, the skin becomes edematous, except where hair follicles cause small indentations of the skin, giving an overall resemblance to orange peel. Shortening of the suspensory ligaments (of Cooper) or retinacula cutis leads to pitting of the overlying skin, pitting that is intensified if the patient raises her arm above her head. Invasion of the pectoralis major by cancer can result in fixation of the breast, seen upon elevation of the ipsilateral limb. Inversion of areolar skin with involvement of the ducts would also be due to involvement of the retinacula cutis.

A 47-year-old woman's right breast exhibited characteristics of peau d'orange, that is, the skin resembled an orange peel. This condition is primarily a result of which of the following? A. Shortening of the suspensory ligaments by cancer in the axillary tail (of Spence) of the breast B. Blockage of cutaneous lymphatic vessels C. Contraction of the retinacula cutis of the areola and nipple D. Invasion of the pectoralis major by the cancer E. Ipsilateral (same side) inversion of the periareolar skin from ductular cancer

Answer C. The thenar muscles (and lumbricals I and II) are innervated by the median nerve, which runs through the carpal tunnel. The carpal tunnel is formed anteriorly by the flexor retinaculum and posteriorly by the carpal bones. Carpal tunnel syndrome is caused by a compression of the median nerve, due to reduced space in the carpal tunnel. The carpal tunnel contains the tendons of flexor pollicis longus, flexor digitorum profundus, and flexor digitorum superficialis muscles and their synovial sheaths. The dorsal interossei, lumbricals III and IV, palmar interossei, and hypothenar muscles are all innervated by the ulnar nerve.

A 48-year-old female court stenographer is admitted to the orthopedic clinic with symptoms of carpal tunnel syndrome, with which she has suffered with for almost a year. Which muscles most typically become weakened in this condition? A. Dorsal interossei B. Lumbricals III and IV C. Thenar D. Palmar interossei E. Hypothenar

C. The recurrent branch of the median nerve innervates the thenar muscles (opponens pollicis, abductor pollicis brevis, and flexor pollicis brevis) and is not responsible for any cutaneous innervation. Damage to the palmar cutaneous branches of the median nerve or to the ulnar nerve would not cause weakness of opposition of the thumb for they are principally sensory in function. The deep branch of the ulnar nerve supplies the hypothenar muscles, adductor and abductor muscles of digits 2-5, and does not innervate the abductor pollicis brevis.

A 48-year-old female piano player visited the outpatient clinic with numbness and tingling in her left hand. A diagnosis was made of nerve compression in the carpal tunnel, and the patient underwent an endoscopic nerve release. Two weeks postoperatively the patient complained of a profound weakness in the thumb, with loss of thumb opposition. The sensation to the hand, however, was unaffected. Which of the following nerves was injured during the operation? A. The first common digital branch of the median nerve B. The second common digital branch of the median nerve C. Recurrent branch of median nerve D. Deep branch of the ulnar nerve E. Anterior interosseus nerve

C. The anterior intercostal arteries are twelve small arteries, two in each of the upper six intercostal spaces at the upper and lower borders. The upper artery lying in each space anastomoses with the posterior intercostal arteries, whereas the lower one usually joins the collateral branch of the posterior intercostal artery. The musculophrenic artery is a terminal branch of the internal thoracic artery (also known as the internal mammary artery), and it supplies the pericardium, diaphragm, and muscles of the abdominal wall. It anastomoses with the deep circumflex iliac artery. The superior epigastric artery is the other terminal branch of the internal thoracic artery, and it supplies the diaphragm, peritoneum, and the anterior abdominal wall and anastomoses with the inferior epigastric artery. The lateral thoracic artery runs along the lateral border of the pectoralis minor muscle and supplies the pectoralis major, pectoralis minor, and serratus anterior. The thoracodorsal artery accompanies the thoracodorsal nerve in supplying the latissimus dorsi muscle and lateral thoracic wall.

A 49-year-old woman who had suffered a myocardial infarction must undergo a bypass graft procedure using the internal thoracic artery. Which vessels will most likely continue to supply blood to the anterior part of the upper intercostal spaces? A. Musculophrenic B. Superior epigastric C. Posterior intercostal D. Lateral thoracic E. Thoracodorsal

Answer C. The radial nerve descends posteriorly between the long and lateral heads of the triceps brachii muscle and passes inferolaterally on the back of the humerus between the medial and lateral heads of the triceps brachii muscle. It eventually enters the anterior compartment and descends to enter the cubital fossa, where it divides into superficial and deep branches. The deep branch of the radial nerve winds laterally around the radius and runs between the two heads of the supinator muscle and continues as the posterior interosseous nerve, innervating extensor muscles of the forearm. Because this injury does not result in loss of sensation over the skin of the upper limb, it is likely that the superficial branch of the radial nerve is not injured. If the radial nerve were injured very proximally, the woman would not be able to extend her elbow. The branches of the radial nerve to the triceps brachii muscle arise proximal to where the nerve runs in the spiral groove. The anterior interosseous nerve arises from the median nerve and supplies the flexor digitorum profundus, flexor pollicis longus, and pronator quadratus muscles, none of which seem to be injured in this example. Injury to the median nerve causes a characteristic flattening (atrophy) of the thenar eminence.

A 52-year-old female band director suffered problems in her right arm several days after strenuous field exercises for a major athletic tournament. Examination in the orthopedic clinic reveals wrist drop and weakness of grasp but normal extension of the elbow joint. There is no loss of sensation in the affected limb. Which nerve was most likely affected? A. Ulnar B. Anterior interosseous C. Posterior interosseous D. Median E. Superficial radial

B. The lunate is the most commonly dislocated carpal bone because of its shape and relatively weak ligaments anteriorly. Dislocations of the scaphoid and triquetrum are relatively rare. The trapezoid and capitate bones are located in the distal row of the carpal bones.

A 54-year-old female marathon runner presents with pain in her right wrist that resulted when she fell with force on her outstretched hand. Radiographic studies indicate an anterior dislocation of a carpal bone (Fig. 6-4). Which of the following bones is most likely dislocated? A. Capitate B. Lunate C. Scaphoid D. Trapezoid E. Triquetrum

E. The anterior interosseous nerve is a branch of the median nerve that supplies the flexor pollicis longus, the lateral half of the flexor digitorum profundus, and the pronator quadratus muscles. If it is injured, flexion of the interphalangeal joint of the thumb will be compromised. The median nerve gives rise to the anterior interosseous nerve but is not a direct enough answer as injury to it would result in more widespread effects. The posterior interosseous nerve supplies extensors in the forearm, not flexors. The radial nerve gives rise to the posterior interosseous nerve and is not associated with the anterior interosseous nerve; therefore, it would not have any effect on the flexors of the forearm. The recurrent median nerve is also a branch of the median nerve but supplies the thenar eminence muscles, and its injury would result in problems with opposable motion of the thumb

A 54-year-old male cotton farmer visits the outpatient clinic because of a penetrating injury to his forearm from a baling hook. After the limb is anesthetized, the site of the wound is opened and flushed thoroughly to remove all debris. The patient is not able to oppose the tip of the thumb to the tip of the index finger, as in making the OK sign. He is able to touch the tips of the ring and little fingers to the pad of his thumb. What nerve has most likely been injured? A. Median B. Posterior interosseous C. Radial D. Recurrent median E. Anterior interosseous

B. The condition described in this patient is called "winging" of the scapula. "Winging" of the scapula occurs when the medial border of the scapula lifts off the chest wall when the patient pushes against resistance, such as a vertical wall. The serratus anterior muscle holds the medial border of the scapula against the chest wall and is innervated by the long thoracic nerve. The serratus anterior assists in abduction of the arm above the horizontal plane by rotating the scapula so that the glenoid fossa is directed more superiorly.

A 55-year-old man is examined in a neighborhood clinic after receiving blunt trauma to his right axilla in a fall. He has difficulty elevating the right arm above the level of his shoulder. Physical examination shows that the inferior angle of his right scapula protrudes more than the lower part of the left scapula. The right scapula protrudes far more when the patient pushes against the examiner's hand with resistance. Which of the following neural structures has most likely been injured? A. The posterior cord of the brachial plexus B. The long thoracic nerve C. The upper trunk of the brachial plexus D. The site of origin of the middle and lower subscapular nerves E. Spinal nerve ventral rami C7, C8, and T1

B. The C6 spinal nerve is primarily responsible for the brachioradialis reflex. C5 and C6 are both involved in the biceps brachii reflex; C5 for motor, C6 for the sensory part of the reflex arc; C7 is the key spinal nerve in the triceps reflex.

A 54-year-old woman was found unconscious on the floor, apparently after a fall. She was admitted to the hospital, and during physical examination it was observed that she had unilateral absence of her brachioradialis reflex. Which spinal nerve is primarily responsible for this reflex in the majority of cases? A. C5 B. C6 C. C7 D. C8 E. T1

D. Normally the distal part of the ulna articulates only with the radius at the distal radioulnar joint at the wrist, a joint that participates in pronation and supination. The head of the ulna does not articulate with any of the carpal bones; instead, it is separated from the triquetrum and lunate bones by the triangular fibrocartilage complex between it and the radius. The pisiform articulates with the triquetrum. The carpal articulation of the radius is primarily that of the scaphoid (the old name is navicular) bone

A 55-year-old female choreographer had been treated in the emergency department after she fell from the stage into the orchestra pit. Radiographs revealed fracture of the styloid process of the ulna. Disruption of the triangular fibrocartilage complex is suspected. With which of the following bones does the ulna normally articulate at the wrist? A. Triquetrum B. Hamate C. Radius and lunate D. Radius E. Pisiform and triquetrum

E. The axillary sheath is a fascial continuation of the prevertebral layer of the deep cervical fascia extending into the axilla. It encloses the nerves of the neurovascular bundle of the upper limb. Superficial fascia is loose connective tissue between the dermis and the deep investing fascia and contains fat, cutaneous vessels, nerves, lymphatics, and glands. The buccopharyngeal fascia covers the buccinator muscles and the pharynx mingles with the pretracheal fascia. The clavipectoral muscle invests the clavicle and pectoralis minor muscle. The axillary fascia is continuous with the pectoral and latissimus dorsi fascia and forms the hollow of the armpit.

A 55-year-old male metallurgist had been diagnosed with carpal tunnel syndrome. To begin the operation, an anesthetic injection into his axillary sheath was given instead of general anesthesia. From which of the following structures does the axillary sheath take origin? A. Superficial fascia of the neck B. Superficial cervical investing fascia C. Buccopharyngeal fascia D. Clavipectoral fascia E. Prevertebral fascia

C. The superior ulnar collateral branch of the brachial artery accompanies the ulnar nerve in its path posterior to the medial epicondyle and is important in the blood supply of the nerve. The profunda brachii artery passes down the arm with the radial nerve. The radial collateral artery arises from the profunda brachii artery and anastomoses with the radial recurrent branch of the radial artery proximal to the elbow laterally. The inferior ulnar collateral artery arises from the brachial artery and accompanies the median nerve into the forearm. The anterior ulnar recurrent artery arises from the ulnar artery and anastomoses with the inferior ulnar collateral artery anterior to the elbow.

A 58-year-old convenience store operator had received a superficial bullet wound to the soft tissues on the medial side of the elbow in an attempted robbery. A major nerve was repaired at the site where it passed behind the medial epicondyle. Bleeding was stopped from an artery that accompanied the nerve in its path toward the epicondyle. Vascular repair was performed on this small artery because of its important role in supplying blood to the nerve. Which of the following arteries was most likely repaired? A. The profunda brachii artery B. The radial collateral artery C. The superior ulnar collateral artery D. The inferior ulnar collateral artery E. The anterior ulnar recurrent artery

C. Adduction of the fifth digit is produced by contraction of the third palmar interosseous muscle. All of the interossei are innervated by the deep branch of the ulnar nerve. Flexion of the proximal interphalangeal joint is a function of the flexor digitorum superficialis, supplied by the median nerve. Opposition of the thumb is a function of the opponens pollicis, supplied by the recurrent branch of the median nerve.

A 60-year-old male butcher accidentally slashed his wrist with his butcher's knife, partially dividing the ulnar nerve. Which of the following actions would most likely be lost as a result of this injury? A. Flexion of the proximal interphalangeal joint of the fifth digit (little finger) B. Extension of the thumb C. Adduction of the fifth digit D. Abduction of the thumb E. Opposition of the thumb

A. The patient has suffered injury to the radial nerve in the midhumeral region. The nerve that provides sensation to the dorsum of the hand proximal to the thumb and index finger is the superficial branch of the radial nerve. The posterior interosseous nerve supplies a strip of skin on the back of the forearm and wrist extensors. The lateral antebrachial cutaneous nerve is a continuation of the musculocutaneous nerve and supplies the lateral side of the forearm. The medial antebrachial cutaneous is a direct branch of the medial cord and supplies skin of the medial side of the forearm. The dorsal cutaneous branch of the ulnar nerve supplies the medial side of the dorsum of the hand.

A 61-year-old man was hit in the midhumeral region of his left arm by a cricket bat. Physical examination reveals an inability to extend the wrist and loss of sensation on a small area of skin on the dorsum of the hand proximal to the first two fingers. What nerve supplies this specific region of the hand? A. Radial B. Posterior interosseous C. Lateral antebrachial cutaneous D. Medial antebrachial cutaneous E. Dorsal cutaneous of ulnar

D. Swan-neck deformity involves slight flexion of the metacarpophalangeal (MCP) joints, hyperextension of the proximal interphalangeal (PIP) joints, and slight flexion of the distal interphalangeal (DIP) joints. This condition results most often from shortening of the tendons of intrinsic muscles, as in rheumatoid arthritis. When asked to straighten the injured finger, the patient is unable to do so and the curvature of the finger somewhat resembles the neck of a swan.

A 67-year-old housepainter visits the outpatient clinic complaining that his hands are getting progressively worse, becoming more and more painful and losing their function. On physical examination of the hands, there is flexion of the metacarpophalangeal joints, extension of the proximal interphalangeal joints, and slight flexion of the distal interphalangeal joints. What is the most likely diagnosis? A. Mallet finger B. Boutonnière deformity C. Dupuytren's contracture D. Swan-neck deformity E. Silver fork wrist deformity

A. The force of the woman's fall on the outstretched hand was transmitted up through the forearm, in this case resulting in dislocation of the olecranon at the elbow, putting traction on the ulnar nerve as it passes around the medial epicondyle of the humerus. Ulnar trauma at the elbow can cause weakness in medial flexion (adduction) at the wrist, from loss of the flexor carpi ulnaris. Ulnar nerve injury also results in sensory loss in the medial hand and paralysis of the interossei and medial two lumbricals, with clawing especially of digits 4 and 5. Injury of the ulnar nerve at the pisiform bone would not affect the flexor carpi ulnaris, nor would it produce sensory loss on the dorsum of the hand because the dorsal cutaneous branch of the ulnar branches off proximal to the wrist. Carpal tunnel problems affect median nerve function, which is not indicated here. The ulnar nerve passes medial to the cubital fossa between the heads of the flexor carpi ulnaris, not between the heads of the flexor digitorum superficialis. Injuries at the radial neck affect the site of division of the radial nerve, and its paralysis would not result in the clinical problems seen in this patient.

A 67-year-old woman had a bad fall while walking her dog the evening before. She states that she fell on her outstretched hand. Radiographs do not demonstrate any bony fractures. The clinician observes the following signs of neurologic injury: weakness of flexion of her wrist in a medial direction, a loss of sensation on the medial side of the hand, and clawing of the fingers. Where is the most likely place of nerve trauma? A. Behind the medial epicondyle B. Between the pisiform bone and the flexor retinaculum C. Within the carpal tunnel D. At the cubital fossa, between the ulnar and radial heads of origin of flexor digitorum superficialis E. At the radial neck, 1 cm distal to the humerocapitellar joint

D. Colles' fracture is a fracture of the distal radius with the distal fragment displaced dorsally. Smith's fracture involves the distal fragment displaced in a volar direction. Smith's fracture is sometimes referred to as a reverse Colles' fracture.

A 68-year-old woman fell when she missed the last step from her motor home. Radiographic examination at the local medical care center reveals a fracture of the distal radius. The distal fragment of the radius is angled forward. What name is commonly applied to this type of injury? A. Colles' fracture B. Scaphoid fracture C. Bennett's fracture D. Smith's fracture E. Boxer's fracture

A. A Colles' fracture is a fracture of the distal end of the radius. The proximal portion of the radius is displaced anteriorly, with the distal bone fragment projecting posteriorly. The displacement of the radius from the wrist often gives the appearance of a dinner fork, thus a Colles' fracture is often referred to as a "dinner fork" deformity. A scaphoid fracture results from a fracture of the scaphoid bone and would thus not cause displacement of the radius. This fracture usually occurs at the narrow aspect ("waist") of the scaphoid bone. Bennett's and boxer's fractures both result from fractures of the metacarpals (first and fifth, respectively). Volkmann's ischemic contracture is a muscular deformity that can follow a supracondylar fracture of the humerus, with arterial laceration into the flexor compartment of the forearm. Ischemia and muscle contracture, with extreme pain, accompany this fracture.

A 68-year-old woman is examined by the senior resident in emergency medicine after she fell on a wet bathroom floor in the shopping center. Physical examination reveals a posterior displacement of the left distal wrist and hand. Radiographic examination reveals an oblique fracture of the radius. Which of the following is the most likely fracture involved in this case? A. Colles' fracture B. Scaphoid fracture C. Bennett's fracture D. Volkmann's ischemic contracture E. Boxer's fracture

B. Dupuytren's contracture or deformity is a result of fibromatosis of palmar fascia, resulting in irregular thickening of the fascial attachments to the skin, which causes gradual contraction of the digits, especially digits 4 and 5. In 50% of cases, it is bilateral in occurrence. Ape hand, or flat hand, is a result of loss of the median and ulnar nerves at the elbow, with paralysis of all long flexors of the fingers and all intrinsic hand muscles. The term can also be specific for just median nerve injury and a flattened thenar eminence. Claw hand results from paralysis of interphalangeal joint extension by interossei and lumbricals, innervated primarily by the ulnar nerve. Wrist drop occurs with radial nerve paralysis and loss of the extensors carpi radialis longus and brevis. Mallet finger results from detachment of the extensor mechanism from the distal phalanx of a finger and unopposed flexion of that distal interphalangeal joint.

A 72-year-old man consulted his physician because he had noticed a thickening of the skin at the base of his left ring finger during the preceding 3 months. As he described it, "There appears to be some hard tissue that is pulling my little and ring fingers into my palm." On examination of the palms of both hands, localized and firm ridges are observed in the palmar skin that extend from the middle part of the palm to the base of the ring and little fingers. What is the medical term for this sign? A. Ape hand B. Dupuytren's contracture C. Claw hand D. Wrist drop E. Mallet finger

B. The fracture line of the upper third of the radius lies between the bony attachments of the supinator and the pronator teres muscles. The distal radial fragment and hand are pronated due to unopposed contraction of pronator teres and pronator quadratus muscles. The proximal fragment deviates laterally by the unopposed contraction of the supinator muscle. The brachioradialis inserts distally on the radius. The brachialis inserts on the coronoid process of the ulna and would not be involved in the lateral deviation of the radius.

A 74-year-old woman is admitted to the emergency department after stumbling over her pet dog. Radiographic examination reveals a fracture of the upper third of the right radius, with the distal fragment of the radius and hand pronated. The proximal end of the fractured radius deviates laterally. Which of the following muscles is primarily responsible for the lateral deviation? A. Pronator teres B. Supinator C. Pronator quadratus D. Brachioradialis E. Brachialis

A. The median nerve supplies sensory innervation to the thumb, index, and middle fingers and also to the lateral half of the ring finger. The median nerve also provides motor innervation to muscles of the thenar eminence. Compression of the median nerve in the carpal tunnel explains these deficits in conjunction with normal functioning of the flexor compartment of the forearm because these muscles are innervated by the median nerve proximal to the carpal tunnel. Also, sensory innervation in the proximal palm will be normal because the palmar branch of the radial nerve usually branches off proximal to the flexor retinaculum. The ulnar nerve is not implicated in these symptoms. It does not provide sensation to digits 1 to 3. Compression of the brachial plexus could not be attributed to pressure from the triceps brachii because this muscle is located distal to the plexus. In addition, brachial plexus symptoms would include other upper limb deficits, rather than the focal symptoms described in this case. Osteoarthritis of the cervical spine would also lead to increasing complexity of symptoms.

A 79-year-old man has numbness in the middle three digits of his right hand and finds it difficult to grasp objects with that hand. He states that he retired 9 years earlier, after working as a carpenter for 50 years. He has atrophy of the thenar eminence (Fig. 6-3). Which of the following conditions is the most likely cause of the problems in his hand? A. Compression of the median nerve in the carpal tunnel B. Formation of the osteophytes that compress the ulnar nerve at the medial epicondyle C. Hypertrophy of the triceps brachii muscle compressing the brachial plexus D. Osteoarthritis of the cervical spine E. Repeated trauma to the ulnar nerve

C. The axillary nerve passes dorsally around the surgical neck of the humerus (accompanied by the posterior circumflex humeral artery) and can be injured when the humerus is fractured at that location. The axillary nerve provides sensation to the skin over the upper, lateral aspect of the shoulder. Therefore, although the patient might not be able to abduct the arm because of the injury, a simple test of skin sensation can indicate whether there is associated nerve injury of the axillary nerve (CN XI). Shrugging the shoulders can help assess trapezius function, thereby testing the spinal accessory nerve. Intact sensation of the skin on the medial aspect of the axilla and arm is an indication that the radial and intercostobrachial nerves are functional. Pushing against an immovable object tests the serratus anterior muscle and the long thoracic nerve

A male skier had a painful fall against a rocky ledge. Radiographic findings revealed a hairline fracture of the surgical neck of the humerus. The third-year medical student assigned to this patient was asked to determine whether there was injury to the nerve associated with the area of injury. Which of the following tests would be best for checking the status of the nerve? A. Have the patient abduct the limb while holding a 10 lb weight B. Have the patient shrug the shoulders C. Test for presence of skin sensation over the lateral side of the shoulder D. Test for normal sensation over the medial skin of the axilla E. Have the patient push against an immovable object like a wall and assess the position of the scapula

A. The anular ligament is a fibrous band that encircles the head of the radius, forming a collar that fuses with the radial collateral ligament and articular capsule of the elbow. The anular ligament functions to prevent displacement of the head of the radius from its socket. In a child of this age the head of the radius is almost the same diameter as the shaft of the bone, so the head is relatively easy to dislocate. The joint capsule functions to allow free rotation of the joint and does not function in its stabilization. The interosseous membrane is a fibrous layer between the radius and ulna helping to hold these two bones together. The radial collateral ligament extends from the lateral epicondyle to the margins of the radial notch of the ulnar and the anular ligament of the radius. The ulnar collateral ligament is triangular ligament and extends from the medial epicondyle to the olecranon of the

A mother tugs violently on her 4-year-old boy's hand to pull him out of the way of an oncoming car and the child screams in pain. Thereafter, it becomes obvious that the child cannot straighten his forearm at the elbow. When the child is seen in the emergency department, radiographic examination reveals a dislocation of the head of the radius. Which of the following ligaments is most likely directly associated with this injury? A. Anular B. Joint capsular C. Interosseous D. Radial collateral E. Ulnar collateral

E. The injury being described is also known as Erb-Duchenne paralysis or "waiter's tip hand" and is relatively common in children after a difficult delivery. This usually results from an injury to the upper trunk of the brachial plexus, presenting with loss of abduction, flexion, and lateral rotation of the arm. The superior trunk of the brachial plexus consists of spinal nerve ventral rami C5-6.

After a forceps delivery of an infant boy, the baby presents with his left upper limb adducted, internally rotated, and flexed at the wrist. The startle reflex is not seen on the ipsilateral side. Which part of the brachial plexus was most likely injured during this difficult delivery? A. Lateral cord B. Medial cord C. Ventral rami of the lower trunk D. Ventral ramus of the middle trunk E. Ventral rami of the upper trunk

Answer B. A supracondylar fracture often results in injury to the median nerve. The course of the median nerve is anterolateral, and at the elbow it lies medial to the brachial artery on the brachialis muscle. The axillary nerve passes posteriorly through the quadrangular space, accompanied by the posterior circumflex humeral artery, and winds around the surgical neck of the humerus. Injury to the surgical neck may damage the axillary nerve. The musculocutaneous nerve pierces the coracobrachialis muscle and descends between the biceps brachii and brachialis muscle. It continues into the forearm as the lateral antebrachial cutaneous nerve. The ulnar nerve descends behind the medial epicondyle in its groove and is easily injured and produces "funny bone" symptoms.

An 18-year-old man is brought to the emergency department after an injury while playing rugby. Imaging reveals a transverse fracture of the humerus about 1 inch proximal to the epicondyles. Which nerve is most frequently injured by the jagged edges of the broken bone at this location? A. Axillary B. Median C. Musculocutaneous D. Radial E. Ulnar

D. The interossei are the most important muscles in extension of the interphalangeal (IP) joints because of the manner of their insertion into the extensor expansion of the fingers, which passes dorsal to the transverse axes of these joints. The lumbrical muscles assist in IP extension, in addition to flexing the metacarpophalangeal joints. Ulnar nerve injury at the wrist results in paralysis of all the interossei and the medial two lumbricals. Extensors of the metacarpophalangeal (MCP) joints are innervated by the deep radial nerve. Unopposed extension of the MCP joints causes them to be held in extension whereas unopposed long flexors of the fingers (supplied by median and ulnar nerves proximally in the forearm) cause them to be flexed into the "claw" position. The lumbricals of digits two and three are still intact because they are supplied by the median nerve, so clawing is not seen as much on these digits. Loss of opposition would result from median or recurrent nerve paralysis. If the ulnar nerve is cut at the wrist, its dorsal cutaneous branch to the dorsum of the hand is unaffected.

An 18-year-old man suffered a significant laceration through the skin and underlying tissues at the distal crease of the wrist. The medical student rotating through the emergency department suspected (correctly) that the ulnar nerve was cut completely through at this location. Which of the following would most likely occur? A. The patient could not touch the tip of the thumb to the tips of the other digits B. There would be loss of sensation on the dorsum of the medial side of the hand C. The patient would be unable to flex the interphalangeal joints D. There would be decreased ability to extend the interphalangeal joints E. There would be no serious functional problem at all to the patient

E. The suprascapular artery passes over, and the suprascapular nerve passes under, the superior transverse scapular ligament. This ligament bridges the suprascapular notch in the upper border of the scapula, converting the notch to foramen. The artery and nerve then pass deep to the supraspinatus muscle, thereafter supplying it and then passing through the spinoglenoid notch to supply the infraspinatus. The subscapular artery is a branch of the third part of the axillary artery; it divides into circumflex scapular and thoracodorsal branches. The transverse cervical artery courses anterior to this site. The dorsal scapular artery and nerve pass deep to the medial border of the scapula. The posterior circumflex humeral branch of the axillary artery passes through the quadrangular space with the axillary nerve.

An orthopedic surgeon exposed a muscle in the supraspinous fossa so that she could move it laterally while repairing an injured rotator cuff. As she reflected the muscle from its bed, an artery was exposed crossing the ligament that bridges the notch in the superior border of the scapula. What artery was this? A. Subscapular B. Transverse cervical C. Dorsal scapular D. Posterior circumflex humeral E. Suprascapular

C. The tendon of the long head of the biceps brachii muscle passes through the glenohumeral joint, surrounded by synovial membrane. The glenohumeral is a ligament that attaches to the glenoid labrum. The long head of the triceps brachii arises from the infraglenoid tubercle, beneath the glenoid fossa. The infraspinatus tendon passes posterior to the head of the humerus to insert on the greater tubercle. The coracobrachialis arises from the coracoid process and inserts on the humerus.

Arthroscopic examination of the shoulder of a 62-year-old woman clearly demonstrated erosion of the tendon within the glenohumeral joint. What tendon was this? A. Glenohumeral B. Long head of triceps brachii C. Long head of biceps brachii D. Infraspinatus E. Coracobrachialis

E. The medial cord has been injured by traction on the lower trunk of the brachial plexus. The medial cord is the continuation of the inferior (lower) trunk of the brachial plexus, which is formed by C8 and T1. C5 and C6 are typically associated with the superior (upper) trunk level and thus the lateral cord. C7 forms the middle trunk. An injury to the posterior cord would usually involve the C7 spinal nerve. This is a typical Klumpke paralysis.

As she fell from the uneven parallel bars, a 17-year-old female gymnast grasped the lower bar briefly with one hand but then fell painfully to the floor. An MRI examination reveals an injury to the medial cord of the brachial plexus. Which of the following spinal nerve levels would most likely be affected? A. C5, C6 B. C6, C7 C. C7, C8 D. C7, C8, T1 E. C8, T1

E. Scoliosis (severe lateral curvature of the spine) in the patient is causing compression or stretching of the T1 spinal nerve ramus by the first rib as the nerve ascends to join C8 and form the lower trunk of the brachial plexus. T1 provides sensation for the medial side of the forearm, via the medial antebrachial cutaneous nerve from the medial cord of the brachial plexus. T1 is the principal source of motor supply to all of the intrinsic muscles in the palm. Its dysfunction affects all fine motor movements of the digits. Long flexors of the fingers are intact; therefore, the median nerve and ulnar nerve are not injured. The extensors of the wrist are functional; therefore, the radial nerve is not paralyzed. The only sensory disturbance is that of the T1 dermatome.

Fine motor function in the right hand of a 14-yearold girl with scoliosis since birth appeared to be quite reduced, including opposition of the thumb, abduction and adduction of the digits, and interphalangeal joint extension. Radiography confirmed that her severe scoliosis was causing marked elevation of the right first rib. Long flexor muscles of the hand and long extensors of the wrist appear to be functioning within normal limits. There is notable anesthesia of the skin on the medial side of the forearm; otherwise, sensory function in the limb is intact. Which of the following neural structures is most likely impaired? A. Median nerve B. Middle trunk of the brachial plexus C. Radial nerve D. Lower trunk of the brachial plexus E. T1 ventral ramus

E. The deep branch of the radial nerve courses between the two heads of the supinator muscle and is located just medial and distal to the lateral epicondyle. After the nerve emerges from the supinator it is called the posterior interosseous nerve. It can be irritated by hypertrophy of the supinator, which compresses the nerve, causing pain and weakness. The ulnar nerve courses laterally behind the medial epicondyle and continues anterior to the flexor carpi ulnaris muscle. The median nerve passes into the forearm flexor compartment; the superficial radial nerve courses down the lateral aspect of the posterior forearm and would not cause pain due to pressure applied to the posterior forearm.

Following several days of 12-hour daily rehearsals of the symphony orchestra for a performance of a Wagnerian opera, the 52-year-old male conductor experienced such excruciating pain in the posterior aspect of his right forearm that he could no longer direct the musicians. When the maestro's forearm was palpated 2 cm distal and posteromedial to the lateral epicondyle, the resulting excruciating pain caused him to grimace. Injections of steroids and rest were recommended to ease the pain. Which of the following injuries is most likely? A. Compression of the ulnar nerve by the flexor carpi ulnaris B. Compression of the median nerve by the pronator teres C. Compression of the median nerve by the flexor digitorum superficialis D. Compression of the superficial radial nerve by the brachioradialis E. Compression of the deep radial nerve by the supinator

E. The extensor tendons of the fingers insert distally on the distal phalanx of each digit. If the tendon is avulsed, or the proximal part of the distal phalanx is detached, the distal interphalangeal joint (DIP) is pulled into total flexion by the unopposed flexor digitorum profundus muscle. This result gives the digit the appearance of a mallet. In boutonnière deformity, the central portion of the extensor tendon expansion is torn over the proximal interphalangeal (PIP) joint, allowing the tendon to move palmarward, causing the tendon to act as a flexor of the PIP joint. This causes the DIP joint to be hyperextended. Swanneck deformity involves slight flexion of the metacarpophalangeal (MCP) joints, hyperextension of PIP joints, and slight flexion of DIP joints. This condition results most often from shortening of the tendons of intrinsic muscles, as in rheumatoid arthritis. Dupuytren's contracture results from connective tissue disorder in the palm, usually causing irreversible flexion of digits 4 and 5. Claw hand occurs with lesions to the median and ulnar nerves at the wrist. In this clinical problem all intrinsic muscles are paralyzed, including the extensors of the interphalangeal joints. The MCP joint extensors, supplied by the radial nerve, and the long flexors of the fingers, supplied more proximally in the forearm by the median and ulnar nerves, are intact and are unopposed, pulling the fingers into the "claw" appearance.

It was reported by the sports media that the outstanding 27-year-old shortstop for a New York team would miss a number of baseball games. He was hit on a fingertip while attempting to catch a ball barehanded. A tendon had been torn. The team doctor commented that the ballplayer could not straighten the last joint of the long finger of his right hand, and the finger would require surgery. From what injury did the ballplayer suffer? A. Claw hand deformity B. Boutonnière deformity C. Swan-neck deformity D. Dupuytren's contracture E. Mallet finger

C. The long thoracic nerve was injured during the axillary dissection, resulting in paralysis of the serratus anterior muscle. The serratus anterior is important in rotation of the scapula in raising the arm above the level of the shoulder. Its loss results in protrusion of the medial border ("winging" of the scapula), which is more obvious when one pushes against resistance. The long thoracic nerve arises from the ventral rami of C5, C6, and C7. The upper trunk (C5, C6) supplies rotator and abductor muscles of the shoulder and elbow flexors. The posterior division of the middle trunk contains C7 fibers for distribution to extensor muscles; likewise, the posterior cord supplies extensors of the arm, forearm, and hand. The lateral cord (C5, C6, and C7) gives origin to the lateral pectoral nerve, the musculocutaneous nerve, and the lateral root of the median nerve. There is no sensory loss in the limb in this patient; injury to any of the other nerve elements listed here would be associated with specific dermatome losses.

Several weeks after surgical dissection of her left axilla for the removal of lymph nodes for staging and treatment of her breast cancer, a 32-year-old woman was told by her general physician that she had "winging" of her left scapula when she pushed against resistance during her physical examination. She told the physician that she had also experienced difficulty lately in raising her left arm above her head when she was combing her hair. In a subsequent consult visit with her surgeon, she was told that a nerve was accidentally injured during the diagnostic surgical procedure and that this produced her scapular abnormality and inability to raise her arm normally. What was the origin of this nerve?' A. The upper trunk of her brachial plexus B. The posterior division of the middle trunk C. Ventral rami of the brachial plexus D. The posterior cord of the brachial plexus E. The lateral cord of the brachial plexus

A. The anterior axillary (or anterior pectoral) nodes are the first lymph nodes to receive most of the lymph from the breast parenchyma, areola, and nipple. From there, lymph flows through central axillary, apical, and supraclavicular nodes in sequence. The interpectoral Rotter's nodes lie between the pectoral muscles and are, unfortunately, an alternate route in some patients, speeding the rate of metastasis. The parasternal nodes receive lymph from the medial part of the breast and lie along the internal thoracic artery and vein.

The 35-year-old woman has a hard breast nodule about 1 cm in diameter slightly above and lateral to the right areola of her right breast. A specific dye is injected into the tissue around the tumor, and an incision is made to expose the lymphatic vessels draining the area, for the lymphatic vessels to take up the dye, which is visible to the eye. The vessels can then be traced to surgically expose the lymph nodes receiving the lymph from the tumor. Which of the following nodes will most likely first encounter the lymph from the tumor? A. Anterior axillary (pectoral) nodes B. Rotter's interpectoral nodes C. Parasternal nodes along the internal thoracic artery and vein D. Central axillary nodes E. Apical or infraclavicular nodes

A. The basilic vein can be used for dialysis, especially when the cephalic vein is judged to be too small, as in this case. The basilic vein can be elevated from its position as it passes through the fascia on the medial side of the arm (brachium). The cephalic vein passes more laterally up the limb. The lateral cubital vein is a tributary to the cephalic vein, and the medial cubital vein joins the basilic vein, both of which are rather superficial in position. The medial antebrachial vein courses up the midline of the forearm (antebrachium) ventrally.

The kidneys of a 32-year-old woman were failing, and she needed to be placed on dialysis. However, the search in her upper limb for a suitable vein was unexpectedly difficult. The major vein on the lateral side of the arm was too small; others were too delicate. Finally, a vein was found on the medial side of the arm that passed through the superficial and deep fascia to join veins beside the brachial artery. Which of the following veins was this? A. Basilic B. Lateral cubital C. Cephalic D. Medial cubital E. Medial antebrachial

C. The first branch of the lateral pectoral nerve is typically the only source of motor supply to the clavicular head of the pectoralis major muscle. If it is injured (as in this case of an iatrogenic injury when the infraclavicular nodes were removed), this part of the muscle undergoes atrophy, leaving an infraclavicular cosmetic deficit. The remainder of the lateral pectoral nerve joins the medial pectoral nerve in a neural arch that provides motor supply to the remaining parts of the pectoralis major and the pectoralis minor. Physical examination reveals no obvious motor or sensory deficits. Loss of the medial pectoral nerve would have no effect on the clavicular head of pectoralis major and might not be discernible. Injury to the lateral cord would lead to loss not only of all of the lateral pectoral nerve but also the musculocutaneous nerve, resulting in biceps brachii and brachialis paralysis and lateral antebrachial sensory loss.

The mastectomy procedure on a 52-year-old woman involved excision of the tumor and removal of lymph nodes, including the pectoral, central axillary, and infraclavicular groups. Six months after her mastectomy, the patient complains to her personal physician of an unsightly deep hollow area inferior to the medial half of the clavicle, indicating a significant area of muscle atrophy and loss. She states that the disfigurement has taken place quite gradually since her mastectomy. Physical examination reveals no obvious motor or sensory deficits. What was the most likely cause of the patient's cosmetic problem? A. Part of the pectoralis major muscle was cut and removed in the mastectomy B. The pectoralis minor muscle was removed entirely in the surgery C. A branch of the lateral pectoral nerve was cut D. The medial pectoral nerve was cut E. The lateral cord of the brachial plexus was injured

D. The patient is suffering from subacromial or subdeltoid bursitis. (If the pain on palpation is less when the arm has been elevated to the horizontal, the bursitis may be thought of as being more subacromial, that is, associated more with the supraspinatus tendon perhaps, for such a bursa may be drawn back under the acromion when the limb is abducted.) The subscapular bursa, beneath the subscapularis muscle, would not present as superficial pain. It can communicate with the glenohumeral joint cavity. Inflammation or arthritic changes within the glenohumeral joint present as more generalized shoulder pain than that present here. The teres minor muscle and tendon are located inferior to the point of marked discomfort.

The right shoulder of a 78-year-old woman had become increasingly painful over the past year. Abduction of the right arm caused her to wince from the discomfort. Palpation of the deltoid muscle by the physician produced exquisite pain. Imaging studies reveal intermuscular inflammation extending over the head of the humerus. Which structure was inflamed? A. Subscapular bursa B. Infraspinatus muscle C. Glenohumeral joint cavity D. Subacromial bursa E. Teres minor muscle

C. The injury has occurred just beyond the third part of the axillary artery. The only collateral arterial channel between the third part of the axillary artery and the brachial artery is between the posterior circumflex humeral and the ascending branch of the profunda brachii, and this anastomotic path is often inadequate to supply the arterial needs of the limb. The posterior circumflex humeral arises from the third part of the axillary artery. It typically anastomoses with a variably small, ascending branch of the profunda brachii branch of the brachial artery. The suprascapular artery anastomoses with the circumflex scapular deep to the infraspinatus. The dorsal scapular artery (passing beneath the medial border of the scapula) has no anastomosis with thoracodorsal within the scope of the injury. The lateral thoracic artery has no anastomoses with the brachial artery. The supreme thoracic artery (from first part of axillary) has no helpful anastomoses with the thoracoacromial (second part of axillary)

The shoulder of a 44-year-old deer hunter had been penetrated by a bolt released from a crossbow. The bolt had transected the axillary artery just beyond the origin of the subscapular artery. A compress is placed on the wound with deep pressure. After a clamp is placed on the bleeding artery, thought is given to the anatomy of the vessel. What collateral arterial pathways are available to bypass the site of injury? A. Suprascapular with circumflex scapular artery B. Dorsal scapular with thoracodorsal artery C. Posterior circumflex humeral artery with deep brachial artery D. Lateral thoracic with brachial artery E. Supreme thoracic artery with thoracoacromial artery

Answer C. A midshaft humeral fracture can result in injury to the radial nerve and deep brachial artery because they lie in the spiral groove located in the midshaft. Injury to the median nerve and brachial artery can be caused by a supracondylar fracture that occurs by falling on an outstretched hand and partially flexed elbow. A fracture of the surgical neck of the humerus can injure the axillary nerve and posterior circumflex humeral artery. The suprascapular artery and nerve can be injured in a shoulder dislocation. The long thoracic nerve and lateral thoracic artery may be damaged during a mastectomy procedure.

While walking to his classroom building, a firstyear medical student slipped on the wet pavement and fell against the curb, injuring his right arm. Radiographic images showed a midshaft fracture of the humerus. Which pair of structures was most likely injured at the fracture site? A. Median nerve and brachial artery B. Axillary nerve and posterior circumflex humeral artery C. Radial nerve and deep brachial artery D. Suprascapular nerve and artery E. Long thoracic nerve and lateral thoracic artery


संबंधित स्टडी सेट्स

CoursePoint Chapter 9: Teaching and Counseling

View Set

ECO/365T Week 1 ,2 Practice test

View Set

Chapter 1 and 2 Study Quanatative

View Set

EXAM 3 ch32 1209-1227, 1231-1235, ch25 838-842, 844-852, 859-863, 872-876, ch16 505-508, ch8 245-249, ch31 1192-1206

View Set

Noun Clause Sentence Prescriptions

View Set